Крок 2 - Медицина 2020 2 день (буклет)

1 / 130
Хворий 48-ми років госпіталізований з нападами судом. Багато років страждає на виразку 12-ти палої кишки (ДПК). Протягом останнього місяця спостерігає щоденне блювання, схуд на 20 кг. Об’єктивно: хворий виснажений. В епігастральній ділянці визначається ”шум плеску” Нижня межа шлунка на рівні гребінцевої лінії. Лабораторно: загальний білок - 47 г/л; K - 2,1 ммоль/л, Na -118 ммоль/л, Ca - 1,6 ммоль/л, хлориди - 82 ммоль/л, гематокрит - 64%. Який попередній діагноз? A 48-year-old patient is hospitalized with seizures. He has been suffering from a duodenal ulcer for many years. During the last month, he has been vomiting daily, lost weight 20 kg. Objectively: the patient is exhausted. In the epigastric region, a 'slap noise' is detected. The lower border of the stomach is at the level of the comb line. Laboratory: total protein - 47 g/l, K - 2.1 mmol/l, Na - 118 mmol/ l, Ca - 1.6 mmol/l, chlorides - 82 mmol/l, hematocrit - 64%. What is the previous diagnosis?

Кровоточива виразка 12-ти палої кишки Bleeding duodenal ulcer

Декомпенсований стеноз воротаря Decompensated gatekeeper stenosis

Субкомпенсований стеноз воротаря Undercompensated gatekeeper stenosis

Компенсований стеноз воротаря Compensated stenosis of the goalkeeper

Малігнізована виразка шлунка Malignant gastric ulcer

2 / 130
Хворий 30-ти років працює клепальником протягом 6-ти років. Скарги на різкий біль у плечовому поясі, особливо в нічний час. Пальці рук німіють та біліють при охолодженні. Став дратівливим. Об’єктивно: кисті набряклі, холодні, ціанотичні. Різко знижені всі види чутливості (больова, температурна, вібраційна). При пробі на холод - симптом 'мертвих пальців’.’ Відзначається слабкість у привідних м’язах V пальця; змінена електрозбудженість. Сухожилкові та періостальні рефлекси живі. Який попередній діагноз? A 30-year-old patient has been working as a riveter for 6 years. Complains of sharp pain in the shoulder girdle, especially at night. The fingers are numb and turn white when cooled . Became irritable. Objectively: the hands are swollen, cold, cyanotic. All types of sensitivity (pain, temperature, vibration) are sharply reduced. When tested for cold - the symptom of 'dead fingers.' There is weakness in the adductor muscles of the V finger; electrical excitability has changed. Tendon and periosteal reflexes are alive. What is the previous diagnosis?

Вібраційна хвороба локальної дії Vibration disease of local effect

Вібраційна хвороба загальної дії Vibration disease of general effect

Вегетативно-сенсорна полінейропатія Vegetative-sensory polyneuropathy

Синдром Рейно Raynaud's syndrome

Вузликовий периартеріїт Nodular periarteritis

3 / 130
У хворого 33-х років біль у грудному відділі хребта з корінцевим синдромом, підвищення температури тіла до 38-39°С упродовж 2-х місяців. Об’єктивно: біль при пальпації у проекції остистих відростків Th9-10. Комп’ютерна томографія грудного відділу: у Th9-10 множинні деструкції, які містять кісткові секвестри, навколо хребців візуалізується паравер-тебральний натічний абсцес. Проба Манту з 2 ТО - папула 19 мм. Вкажіть найбільш імовірний діагноз: A 33-year-old patient has pain in the thoracic spine with radicular syndrome, an increase in body temperature to 38-39°C for 2 months. Objectively : pain during palpation in the projection of the spinous processes of Th9-10. Computed tomography of the chest: in Th9-10 multiple destructions containing bone sequestrations, a paravertebral natic abscess is visualized around the vertebrae. Mantoux test with 2 TO - papule 19 mm. Specify the most likely diagnosis:

Травматичний спондиліт Traumatic spondylitis

Туберкульозний спондиліт Tuberculous spondylitis

Анкілозуючий спондилоартрит Ankylosing spondylitis

Метастатичне ураження хребта Metastatic lesion of the spine

Неспецифічний остеомієліт хребта Nonspecific osteomyelitis of the spine

4 / 130
Дівчинка 3-х місяців. Захворіла гостро, скарги на підвищення температури до 38°С, сухий кашель, задишку. При огляді стан тяжкий. Відзначається втягнений податливих місць грудної клітки, роздування крил носа, періоральний ціаноз. ЧД- 60/хПеркуторно: легеневий звук з коробковим відтінком. Аускультативно: маса дрібнопухирцевих хрипів. Аналіз крові: НЬ- 118 г/л, еритроцити -4,3 • 1012/л, лейкоцити - 7,3 • 109/л, ШOE-15 мм/год. Рентгенографія грудної клітки: підвищена прозорість легеневих полів, низьке стояння діафрагми. Вкажіть діагноз: A 3-month-old girl. She became acutely ill, complained of an increase in temperature up to 38°C, a dry cough, shortness of breath. On examination, the condition is severe. There is an indentation of the pliable areas of the chest cages, swelling of the wings of the nose, perioral cyanosis. ChD - 60/x Percussion: lung sound with a boxy tone. Auscultation: a mass of small alveolar rales. Blood analysis: Hb - 118 g/l, erythrocytes -4.3 • 1012/l, leukocytes - 7 ,3 • 109/l, SOE-15 mm/h. X-ray of the chest: increased transparency of the lung fields, low position of the diaphragm. Specify the diagnosis:

Двобічна пневмонія Bilateral pneumonia

Гострий простий бронхіт Acute simple bronchitis

Гострий обструктивний бронхіт Acute obstructive bronchitis

Муковісцидоз Cystic Fibrosis

5 / 130
Породілля на 7 добу післяпологового періоду надійшла в гінекологічне відділення з діагнозом післяпологовий ендоміометрит, лохіометрит. Жінку турбує біль внизу живота, підвищення температури до 38,7°С, мутні кров’янисто-серозні виділення зі статевих шляхів. Після вакуум-аспірації вмісту матки відзначено утруднення дихання частотою до 40/хв., акроціаноз, AT- 85/50 мм рт.ст. Пульс - 105/хв. Який попередній діагноз можна встановити? On the 7th day of the postpartum period, a woman in labor was admitted to the gynecological department with a diagnosis of postpartum endomyometritis, lochiometritis. The woman is worried about pain in the lower abdomen, an increase in temperature up to 38.7°C, cloudy bloody-serous secretions from the genital tract. After vacuum aspiration of the contents of the uterus, breathing difficulties were noted with a frequency of up to 40/min., acrocyanosis, AT- 85/50 mm Hg. Pulse - 105/min. What preliminary diagnosis can be established?

Сепсис Sepsis

Септичний шок Septic shock

Синдром системної запальної відповіді Systemic inflammatory response syndrome

Перфорація матки Uterine perforation

Метротромбофлебіт Metrothrombophlebitis

6 / 130
У чоловіка 55-ти років 2 роки тому верифіковано цироз печінки. Впродовж 2-х тижнів стан погіршився, з’явились сонливість, запаморочення, втрата орієнтації у просторі і часі. Напередодні відзначав день народження в ресторані. Який синдром зумовив погіршення стану хворого? A 55-year-old man was diagnosed with liver cirrhosis 2 years ago. Over the course of 2 weeks, his condition worsened, drowsiness, dizziness, loss of orientation in space and times. He celebrated his birthday in a restaurant the day before. What syndrome caused the patient's condition to deteriorate?

Гіперспленізму Hypersplenism

Холестазу Cholestasis

Жовтяниці Jaundice

Печінкової енцефалопатії Hepatic encephalopathy

Цитолізу Cytolysis

7 / 130
Чоловiк 32-х рокiв госпiталiзований на 5-й день хвороби зi скаргами на пiдвищення температури до 38,4oC, загальну слабкiсть, появу в лiвiй пахвовiй дiлянцi щiльного утвору 5х6 см, малоболючого, рухомого. Шкiра над утвором не змiнена. Обличчя хворого гiперемоване, пастозне. Частота серцевих скорочень - 95/хв., артерiальний тиск - 100/80 мм рт.ст. В загальному аналiзi кровi - помiрна лейкопенiя. Працює ветсанiтаром. Який дiагноз є найбiльш iмовiрним? A 32-year-old man was hospitalized on the 5th day of illness with complaints of an increase in temperature to 38.4oC, general weakness, and the appearance of a 5x6 thick mass in the left axillary area cm, painless, mobile. The skin over the lesion is not changed. The patient's face is hyperconvulsive, pasty. Heart rate - 95/min., arterial pressure - 100/80 mm Hg. In the general blood analysis - moderate leukopenia. He works as a vet. What is the diagnosis most likely?

Iєрсинiоз Yersiniosis

Туляремiя Tularemia

Чума Plague

Доброякiсний лiмфоретикульоз Benign lymphoreticulosis

Гiдраденiт Hydradenite

8 / 130
Хвора 50-ти років - рентгенлаборант зі стажем роботи у відділенні променевої терапії 12 років. Скаржиться на біль в серці, шлунку, слабкість, втомлюваність, поганий сон, головний біль, запаморочення. Аналіз крові: лейкоцити - 3,8 Г/л, лімфопенія, тромбоцити - 200 Г/л. Найбільш імовірний діагноз? The patient is 50 years old - an X-ray technician with 12 years of experience in the radiation therapy department. She complains of pain in the heart, stomach, weakness, fatigue, bad sleep, headache pain, dizziness. Blood analysis: leukocytes - 3.8 G/l, lymphopenia, platelets - 200 G/l. The most likely diagnosis?

Хронічна променева хвороба Chronic radiation sickness

Клімактеричний розлад Climacteric Disorder

Вегето-судинна дистонія Vegeto-vascular dystonia

Синдром хронічної втоми Chronic fatigue syndrome

Ішемічна хвороба серця Ischemic heart disease

9 / 130
Хворий 58-ми років скаржиться на слабкість, набряки ніг, задишку, анорексію. Протягом багатьох років хворіє на хронічний бронхіт. Останні 5 років відзначає посилення виділення харкотиння, часто гнійного характеру. Об’єктивно: ЧСС- 80/хв, АТ-120/80 ммрт.ст., розповсюджені набряки, шкіра бліда, суха, тургор знижений. У сечі: значна протеїнурія, циліндрурія. Вкажіть найбільш вірогідний патологічний процес у нирках: A 58-year-old patient complains of weakness, swelling of the legs, shortness of breath, anorexia. He has been suffering from chronic bronchitis for many years. In the last 5 years, he has noted an increase in sputum, often of a purulent nature. Objectively: heart rate - 80/min, blood pressure - 120/80 mmHg, widespread edema, pale, dry skin, reduced turgor. In the urine: significant proteinuria, cylindruria. Indicate the most likely pathological process in the kidneys:

Амілоїдоз нирок Kidney amyloidosis

Хронічний пієлонефрит Chronic pyelonephritis

Гострий гломерулонефрит Acute glomerulonephritis

Інтерстиціальний нефрит Interstitial jade

Хронічний гломерулонефрит Chronic glomerulonephritis

10 / 130
Жінка 46 років виявила вузол в ділянці щитоподібної залози розміром 2-3 см, який зміщується при ковтанні, з чіткими межами. При скануванні виявлено холодний вузол в правій частці щитоподібної залози. Лімфовузли шиї не збільшені. Яка лікувальна тактика? A 46-year-old woman discovered a 2-3 cm nodule in the area of the thyroid gland, which shifts when swallowing, with clear borders. During the scan, a cold nodule was found in the right lobe of the thyroid gland glands. The lymph nodes of the neck are not enlarged. What are the treatment tactics?

Променева терапія Radiotherapy

Правобічна гемітиреоїдсктомія з терміновим гістологічним дослідженням Right-sided hemithyroidectomy with urgent histological examination

Геміструмектомія Hemistrumectomy

Геміструмектомія з висіченням клітковини шиї за Крайлем Hemistrumectomy with neck tissue excision according to Crail

Спостереження Observations

11 / 130
Хворий 45-ти років скаржиться на біль по внутрішній поверхні гомілки і стегна справа. Хворіє протягом 5 діб, коли після фізичного навантаження з’явився біль у гомілці, який поступово поширився на стегно. Об’єктивно: у проекції великої підшкірної вени на гомілці та стегні спостерігається гіперемія шкіри і припухлість, при пальпації - ущільнення та болючість. Набряки правої нижньої кінцівки відсутні, пульсація артерій на стопах визначається. Яка найбільш імовірна патологія, що зумовлює таку картину? A 45-year-old patient complains of pain on the inner surface of the lower leg and right thigh. He has been ill for 5 days, when after physical exertion pain appeared in the lower leg, which gradually spread to the thigh. Objectively: in the projection of the large saphenous vein on the lower leg and thigh, there is hyperemia and swelling of the skin, on palpation - compaction and tenderness. There is no swelling of the right lower limb, pulsation of the arteries on the feet is determined. What is the most likely pathology that causes such a picture?

Висхідний тромбофлебіт Ascending thrombophlebitis

Гострий тромбоз підколінної та стегнової артерій Acute thrombosis of popliteal and femoral arteries

Спонтанний розрив великої підшкірної вени Spontaneous rupture of the great saphenous vein

Гострий лімфангіт Acute lymphangitis

Гострий тромбоз підколінної та стегнових вен Acute thrombosis of popliteal and femoral veins

12 / 130
Департамент охорони здоров’я області розробляє регіональну програму ’’Здоров’я населення області’’. До якого розділу профілактичних заходів слід віднести комплекс заходів запобігання шкідливому впливу природного, виробничого і побутового середовища? The Department of Health of the region is developing the regional program ``Health of the population of the region.'' production and household environment?

Лікувально-оздоровчі Medical and health

Медико-біологічні Medical-biological

Санітарно-епідеміологічні Sanitary and epidemiological

Медико-соціальні Medical-social

Функціонально-біологічні Functional-biological

13 / 130
Чоловік 56-ти років хворіє на ХОЗЛ упродовж 20-ти років. На висоті значного фізичного навантаження відчув зненацька різку змішану задишку, біль у грудній клітці справа, серцебиття. Об’єктивно: стан важкий, AT- 100/70 мм рт.ст., ЧСС- 110/хв., ЧД- 28/хв., права половина грудної клітки відстає в акті дихання. Перкуторно: коробковий звук над правою легенею, аускультативно дихання різко ослабленЯкий невідкладний стан у хворого? A 56-year-old man has been suffering from COPD for 20 years. At the height of significant physical exertion, he suddenly felt a sharp mixed shortness of breath, pain in the chest on the right, palpitations. Objectively: the condition is severe, AT- 100/70 mm Hg, heart rate- 110/min, HR- 28/min, the right half of the chest lags behind in the act of breathing. Percussion: a box sound over the right lung, auscultatively breathing is sharply weakened. What is the patient's urgent condition?

Тромбоемболія легеневої артерії Thromboembolism of the pulmonary artery

Спонтанний пневмоторакс Spontaneous pneumothorax

Напад бронхіальної астми Bronchial asthma attack

Інфаркт міокарда Myocardial infarction

Негоспітальна пневмонія Community-acquired pneumonia

14 / 130
На прийом до уролога звернувся хворий 75-ти років зі скаргами на постійний, ниючий біль в промежині і надлобковій ділянці, в’ялий струмінь сечі, часте, болюче сечовипускання, ніктурію. Хворіє протягом декількох місяців, коли поступово почало утруднюватись сечовипускання, з’явився біль в промежині. При ректальному дослідженні - простата збільшена в розмірах (більше за рахунок правої долі), щільна, асиметрична, центральна борозна згладжена, права доля кам’янистої щільності, безболісна, бугриста. Яку патологію можна припустити у хворого? A 75-year-old patient came to see a urologist with complaints of constant, aching pain in the perineum and suprapubic area, sluggish stream of urine, frequent, painful urination , nocturia. He has been sick for several months, when urination gradually began to become difficult, pain in the perineum appeared. During rectal examination - the prostate is enlarged (larger due to the right lobe), dense, asymmetric, the central furrow is smoothed, the right lobe is stony dense, painless, lumpy. What pathology can be assumed in the patient?

Рак прямої кишки Rectal cancer

Хронічний простатит Chronic prostatitis

Доброякісна гіперплазія парауретральних залоз Benign hyperplasia of paraurethral glands

Рак передміхурової залози Prostate cancer

Рак сечового міхура Bladder Cancer

15 / 130
Чоловiка 42-х рокiв, шахтаря, через 12 годин звiльнено з-пiд завалу. Об’єктивно: обидвi гомiлки i ступнi блiдого кольору. Пульсацiї периферiйних судин немає. Чутливiсть та пасивнi рухи в суглобах вiдсутнi. Доставлений iз джгутами на обох кiнцiвках. Якi невiдкладнi заходи першої лiкарської допомоги необхiдно провести для запобiгання мiоглобiнурiї та гострої ниркової недостатностi? Хiрургiчний профiль 19 A 42-year-old man, a miner, was freed from the rubble after 12 hours. Objectively: both lower legs and feet are pale in color. There is no pulsation of peripheral vessels. Tenderness and passive motion in the joints are absent. Delivered with tourniquets on both ends. What are the immediate first aid measures to be taken to prevent myoglobinuria and acute renal failure? Surgical profile 19

Зняти джгут, дезiнтоксикацiйна терапiя Remove tourniquet, detoxification therapy

Джгут не знiмати, дезiнтоксикацiйна терапiя Do not remove tourniquet, detoxification therapy

Джгут не знiмати, гiпербарична оксигенацiя Do not remove the tourniquet, hyperbaric oxygenation

Джгут не знiмати, ампутацiя кiнцiвок вище джгута Do not remove the tourniquet, amputation of the limbs above the tourniquet

Зняти джгут, гiпербарична оксигенацiя Remove tourniquet, hyperbaric oxygenation

16 / 130
Чоловік 45-ти років, звернувся до ліка¬ря зі скаргами на дратівливість, підвищену втомленість, схуднення, серцебиття, перебої в роботі серця. При пальпації лівої долі щитоподібної залози пальпується утворен¬ня, щільно-еластичної консистенції, безбо¬лісне. яке при ковтанні зміщується разом з щитоподібною залозою. Позитивні очні симптоми, екзофтальм, порушення конвер¬генції. Артеріальний тиск - 135/80 мм рт.ст., пульс - 110/хв. При ультразвуковому дослі¬дженні щитоподібної залози у лівій долі візуалізується гіперехогенне округле утво¬рення розмірами 2x3 см, з чіткими конту¬рами, однорідної структури. Який висновок лікаря після проведеного обстеження буде найбільш коректним? A 45-year-old man consulted a doctor with complaints of irritability, increased fatigue, weight loss, palpitations, heart failure. When palpating the left thyroid lobe gland, a dense, elastic consistency, painless formation is palpable, which, when swallowed, moves together with the thyroid gland. Positive eye symptoms, exophthalmos, convergence disorder. Blood pressure - 135/80 mm Hg, pulse - 110 /min. During the ultrasound examination of the thyroid gland in the left lobe, a hyperechoic rounded formation measuring 2x3 cm, with clear contours and a homogeneous structure is visualized. What conclusion of the doctor after the examination will be the most correct?

Кіста щитоподібної залози Thyroid cyst

Гострий тиреоїдит Acute thyroiditis

Вузол щитоподібної залози Thyroid nodule

Рак щитоподібної залози Thyroid cancer

Дифузно-токсичний зоб Diffuse toxic goiter

17 / 130
Хворий 57-ми років перебуває на лікуванні з приводу політравми. Через 8 діб виникли скарги на виражений кашель з виділенням гнійного харкотиння, виражену задишку, підвищення температури до 38,8°C. В анамнезі: хронічний бронхіт, палить 20 років. Об’єктивно: в легенях вислуховується жорстке дихання, в нижніх відділах правої легені - вологі дрібнопухирчасті хрипи. Проба на ВІЛ - негативна. Який діагноз у даного хворого? A 57-year-old patient is being treated for polytrauma. After 8 days, there were complaints of severe cough with purulent sputum, severe shortness of breath, temperature rise to 38, 8°C. History: chronic bronchitis, smoker for 20 years. Objectively: harsh breathing is heard in the lungs, moist fine-vesicular rales in the lower parts of the right lung. The HIV test is negative. What is the diagnosis of this patient?

Інфекційне загострення бронхіту Infectious exacerbation of bronchitis

Аспіраційна пневмонія Aspiration pneumonia

Позагоспітальна пневмонія Community-acquired pneumonia

Нозокоміальна пневмонія Nosocomial pneumonia

Пневмонія осіб з імунодефіцитом Pneumonia of persons with immunodeficiency

18 / 130
Хворому 65-ти років встановлено діагноз рак головного мозку IV стадії, Чоловік з вираженими епілептичними нападами та больовим синдромом. Який вид медичної допомоги необхідний для полегшення стану хворого? A 65-year-old patient was diagnosed with stage IV brain cancer, a man with severe epileptic seizures and pain syndrome. What type of medical care is needed to alleviate the patient's condition?

Паліативна медична допомога Palliative medical care

Первинна медична допомога Primary medical care

Третинна медична допомога Tertiary medical care

Санаторно-курортна допомога Sanatorium and spa assistance

Вторинна медична допомога Secondary medical care

19 / 130
У новонародженого, який народився на 29 тижні гестації, внаслідок крайового передлежання плаценти, з перших годин життя спостерігається наростання частоти дихальних рухів до 75/хв., виражені втяжіння міжреберних проміжків та мечеподібного відростка в акті дихання, експіраторний стогін, чутний на відстані. Шкірні покриви дитини ціанотичпі. Над легенями вислуховується послаблене дихання та розсіяна крепітація. Який найбільш імовірний діагноз? In a newborn born at the 29th week of gestation, as a result of marginal placenta previa, from the first hours of life, an increase in the frequency of respiratory movements up to 75/min., pronounced intercostal retractions interspaces and xiphoid process in the act of breathing, expiratory groan, audible in the distance. The skin of the child is cyanotic. Over the lungs, weakened breathing and scattered crepitations are heard. What is the most likely diagnosis?

Внутрішньочерепний крововилив в намет мозочка Intracranial hemorrhage in the tent of the cerebellum

Тяжка асфіксія новонароджених Severe asphyxia of newborns

Пізня вроджена пневмонія Late congenital pneumonia

Гострий обструктивний бронхіт Acute obstructive bronchitis

Респіраторний дистрес-синдром Respiratory distress syndrome

20 / 130
При дослідженні гігієнічних умов фізичного виховання школярів у спортивному залі встановлено: середня температура повітря +22° С, відносна вологість повітря - 55%, швидкість руху повітря 0,4 м/с, об’єм вентиляції на одну дитину - 75 м3/год, коефіцієнт природного освітлення - 2,0%, рівень штучного освітлення (з використанням люмінесцентних ламп) - 300 лк. Вкажіть параметр, який необхідно привести у відповідність до гігієнічних вимог: When studying the hygienic conditions of physical education of schoolchildren in the sports hall, the following was established: average air temperature +22° C, relative air humidity - 55%, air movement speed 0.4 m/s, volume of ventilation per child - 75 m3/h, coefficient of natural lighting - 2.0%, level of artificial lighting (using fluorescent lamps) - 300 lux. Specify the parameter that must be brought into compliance with hygienic requirements :

Рівень штучного освітлення Artificial light level

Об’єм вентиляції на одну дитину за годину Volume of ventilation per child per hour

Середня температура повітря Average air temperature

Швидкість руху повітря Air speed

Коефіцієнт природного освітлення Coefficient of natural light

21 / 130
У хворого 43-х років наступного дня після введення антибіотика з приводу ангіни погіршилося самопочуття, підвищилася температура тіла до 39,4оС, з’явилася висипка на шкірі, Об’єктивно: за гальний стан хворого тяжкий, шкіра тулуба й кінцівок різко гіперемоваиа, болюча при пальпації, множинні бульозні елементи та ерозії з відшаруванням епідермісу. Симптом Нікольського - різко позитивний. В яке відділення госпіталізувати хворого? A 43-year-old patient felt worse the day after taking an antibiotic for angina, his body temperature rose to 39.4oC, a skin rash appeared, Ob 'objectively: the general condition of the patient is severe, the skin of the trunk and limbs is hyperemic, painful on palpation, multiple bullous elements and erosions with peeling of the epidermis. Nikolsky's symptom is strongly positive. In which department should the patient be hospitalized?

Реанімаційне Resuscitation

Дерматологічне Dermatological

Інфекційне Infectious

Терапевтичне Therapeutic

Алергологічне Allergological

22 / 130
Пацієнтка 35-ти років пред’являє скарги на сукровичні виділення з соска лівої молочної залози. При огляді, пальпації - патології не виявлено. Для якого доброякісного захворювання є характерним даний симптом? A 35-year-old patient complains of blood-tinged discharge from the nipple of the left mammary gland. On examination, palpation, no pathology was detected. Which benign disease is characteristic this symptom?

Кіста молочної залози Breast gland cyst

Інфільтративний мастит Infiltrative mastitis

Масталгія Mastalgia

Внутрішньопротокова папілома Intraductal papilloma

Фіброаденома молочної залози Breast fibroadenoma

23 / 130
У хворої 31-го року після проведеної субтотальної резекції щитоподібної залози з приводу токсичного зоба виникли серцебиття, пітливість, психомоторне збудження. Пацієнтка знепритомніла. Об’єктивно: температура тіла - 39,7°С, частота дихання - 38/хв.; ЧСС- 135/хв.^ миготлива аритмія. AT- 190/80 мм рт.ст’ Імовірне ускладнення: A 31-year-old patient developed palpitations, sweating, psychomotor agitation after subtotal resection of the thyroid gland for toxic goiter. The patient fainted. Objectively: body temperature - 39.7°C, respiratory rate - 38/min.; heart rate - 135/min. atrial fibrillation. AT - 190/80 mm Hg Probable complication:

Тиреотоксичний криз Thyrotoxic crisis

Істеричний напад Hysterical attack

Бактеріальний шок Bacterial shock

Рецидив дифузного токсичного зоба Recurrence of diffuse toxic goiter

Гіпертонічний криз Hypertensive crisis

24 / 130
При огляді доношеної новонародженої дівчинки з масою тіла 2600 г у віці 3-х діб відзначаються судоми тонічного характеру, симптом Грефе, горизонтальний ністагм, фебрильна лихоманка, іктеричність шкіри. Велике тім’ячко вибухає, напружене, змішана задишка, у легенях - крепітуючі хрипи з обох сторін, приглушеність серцевих тонів. Поставлено діагноз: сепсис новонароджених. Яке додаткове діагностичне обстеження Ви призначите в першу чергу? When examining a full-term newborn girl with a body weight of 2600 g at the age of 3 days, tonic convulsions, Graefe's symptom, horizontal nystagmus, febrile fever, icterus of the skin are noted. The large head is bulging, strained, mixed shortness of breath, in the lungs - crepitus rales on both sides, muffled heart sounds. The diagnosis is made: sepsis of newborns. What additional diagnostic examination will you prescribe in the first place?

Острофазові показники Acute phase indicators

Моніторинг сатурації кисню Oxygen saturation monitoring

Електрокадіографія Electrocardiography

Люмбальиа пункція Lumbar puncture

Загальний аналіз сечі General urinalysis

25 / 130
Хворий 70-ти років скаржиться на слабкість, запаморочення, короткочасні періоди непритомності, біль в ділянці серця. Під час аускультації серця: частота серцевих скорочень - 40/хв., тони ритмічні, I тон періодично значно посилений, артеріальний тиск -160/90 мм рт.ст. Яка найбільш імовірна причина гемодинамічних порушень? A 70-year-old patient complains of weakness, dizziness, short-term periods of unconsciousness, pain in the heart area. During auscultation of the heart: heart rate - 40/min. , tones are rhythmic, I tone is periodically significantly increased, blood pressure -160/90 mm Hg. What is the most likely cause of hemodynamic disturbances?

Повна блокада лівої ніжки пучка Гіса Complete blockade of the left leg of the bundle of His

АВ блокада I ступеня AV blockade of the 1st degree

Брадисистолічна форма миготливої аритмії Bradysystolic form of atrial fibrillation

Синусова брадикардія Sinus bradycardia

АВ блокада III ступеня AV block III degree

26 / 130
До дерматолога звернувся чоловік віком 43 роки у зв'язку із наявністю висипки на шкірі кистей, яка супроводжується свербежем. Хворіє 2 тижні. Об'єктивно: у вказаних місцях відзначається гіперемія та набряк шкіри, везикули, мокнуття, екскоріації на місцях розчухування. Який найбільш імовірний діагноз у хворого? A 43-year-old man consulted a dermatologist due to the presence of a rash on the skin of the hands, which is accompanied by itching. He has been sick for 2 weeks. Objectively: in the indicated places there is hyperemia and swelling of the skin, vesicles, wetness, excoriations at the places of scratching. What is the most likely diagnosis for the patient?

Дерматит алергічний Allergic dermatitis

Дерматит контактний Contact dermatitis

Екзема справжня Eczema is true

Токсикодермія Toxicoderma

Екзема мікробна Microbial eczema

27 / 130
Жінка 78-ми років скаржиться доньці, що її сусідка систематично краде у неї консервацію з підвалу, намагається її отруїти, розкидаючи по городу ’’отруту”. В якості ’’доказів” показує на добрива, що були раніше завезені сином хворої. Який психопатологічний синдром можна припустити? A 78-year-old woman complains to her daughter that her neighbor systematically steals preserves from her basement, tries to poison her by scattering 'poison' in the garden. As The 'evidence' points to fertilizers that were previously imported by the patient's son. What psychopathological syndrome can be assumed?

Галюцинаторний Hallucinatory

Деліріозний Delirious

Депресивний Depressed

Паранояльний Paranoid

Дементний Demented

28 / 130
При вивченні середнього рівня та характера різноманітності деяких лабораторних показників отримані такі дані: для загального білку крові - середнє квадратичне відхілення ± 4г/л, коефіцієнт варіації 6%; для ШОЕ - відповідно ± 2 мм/год, 23%. Яка з вивчаємих ознак є найбільш різноманітною? When studying the average level and nature of the diversity of some laboratory indicators, the following data were obtained: for total blood protein - mean square deviation ± 4g/l, coefficient of variation 6%; for ESR - respectively ± 2 mm/h, 23%. Which of the studied signs is the most diverse?

Для вивчення різноманітності потрібні додаткові дослідження More research is needed to study diversity

Швидкість осідання еритроцитів (ШОЕ) Erythrocyte Sedimentation Rate (ESR)

Загальний білок сироватки крові Total serum protein

Для вивчення різноманітності потрібні додаткові розрахунки More calculations are required to study diversity

Відмінності в різноманітності ознак відсутні There are no differences in the variety of signs

29 / 130
У вогнищах облисіння на голові волосся обламане на рівні 4-6 мм від поверхні шкіри, відзначається гіперемія, мукоподібне лущення. Імовірний попередній діагноз? In foci of baldness on the head, the hair is broken off at the level of 4-6 mm from the surface of the skin, there is hyperemia, muco-like desquamation. Possible preliminary diagnosis?

Вторинний сифіліс Secondary syphilis

Мікроспорія волосистої частини голови Microsporia of scalp

Себорейне облисіння Seborrheic alopecia

Дискоїдний червоний вовчак Discoid lupus erythematosus

Гніздова плішивість Nest baldness

30 / 130
Чоловік 68-ми років звернувся до лікаря зі скаргами на сильний головний біль, нудоту, відчуття дискомфорту за грудиною. Протягом 2-х років страждає на гіпертонічну хворобу. Свідомість збережена. Ознак неврологічного дефіциту немає. AT- 220/100 мм рт.ст., ЧСС- 92/хв. ЕКГ: ритм синусовий, ознаки гіпертрофії лівого шлуночка. Яке ускладнення розвилося? A 68-year-old man consulted a doctor with complaints of a severe headache, nausea, a feeling of discomfort behind the sternum. He has been suffering from hypertension for 2 years. Consciousness preserved. There are no signs of neurological deficit. AT - 220/100 mm Hg, heart rate - 92/min. ECG: sinus rhythm, signs of left ventricular hypertrophy. What complication developed?

Розшаровуюча аневризма аорти Dissecting aortic aneurysm

Гостре порушення мозкового кровообігу Acute cerebrovascular accident

Інфаркт міокарда Myocardial infarction

Неускладнений гіпертензивний криз Uncomplicated hypertensive crisis

Ускладнений гіпертензивний криз Complicated hypertensive crisis

31 / 130
Хлопчик 2-х років захворів гостро: скарги на підвищення температури до 37,8° С, осиплість голосу, ’’гавкаючий кашель’’ утруднене дихання. Об’єктивно: шкіра бліда, чиста. ЧД- 30/хв., інспіраторна задишка в стані неспокою. Над легенями жорстке дихання, хрипів немає. Перкуторно - легеневий звук не змінений. Рівень Са крові в нормі. В загальному аналізі крові - лейкопенія, лімфоцитоз. В першу чергу можна припустити: A 2-year-old boy became acutely ill: complaints of an increase in temperature up to 37.8°C, hoarseness of voice, 'barking cough', difficulty breathing. About' objectively: the skin is pale, clean. BH - 30/min., inspiratory shortness of breath in a state of restlessness. Hard breathing over the lungs, no wheezes. Percussion - lung sound is not changed. Blood Ca level is normal. In the general blood analysis - leukopenia, lymphocytosis. First of all, we can assume:

Гострий обструктивний бронхіт Acute obstructive bronchitis

Гострий простий бронхіт Acute simple bronchitis

Спазмофілію Spasmophilia

Гострий обструктивний ларинготрахеїт Acute obstructive laryngotracheitis

Бронхіальну астму Bronchial asthma

32 / 130
Жінка 42-х років звернулася до лікаря зі скаргами на м’язову слабкість у верхніх і нижніх кінцівках. Об’єктивно: у періорбітальних ділянках еритема з бузковим відтінком, макульозна еритема на розгинальних поверхнях пальців. В аналізі крові виявлено суттєве підвищення рівня креатинфосфокінази. Яким є імовірний діагноз? A 42-year-old woman consulted a doctor with complaints of muscle weakness in the upper and lower limbs. Objectively: in the periorbital areas erythema with a lilac tint, macular erythema on the extensor surfaces of the fingers. A blood test revealed a significant increase in the level of creatine phosphokinase. What is the probable diagnosis?

Гіпопаратиреоз Hypoparathyroidism

Дерматоміозит Dermatomyositis

Хвороба Кушинга Cushing's disease

Системна склеродермія Systemic scleroderma

Системний червоний вовчак Systemic lupus erythematosus

33 / 130
Хвора 25-ти років вагітність 12 тижнів, госпіталізована у протитуберкульозну лікарню із вперше виявленим інфільтративним туберкульозом верхньої частки лівої легені. Бактеріовиділювач. Відмовилась від abrasio, Який із перерахованих протитуберкульозних препаратів НЕ МОЖНА призначати цій хворій? A 25-year-old patient, 12 weeks pregnant, was hospitalized in an anti-tuberculosis hospital with the first detected infiltrative tuberculosis of the upper lobe of the left lung. Bacteriolator. She refused abrasio, which of the listed anti-tuberculosis drugs drugs CANNOT be prescribed to this patient?

Етамбутол Etambutol

Стрептоміцин Streptomycin

Піразинамід Pyrazinamide

Рифампіцин Rifampicin

Ізоніазид Isoniazid

34 / 130
Пацієнт звернувся у клініку на 2-й день хвороби зі скаргами на слабкість, двоїння предметів, сухість у роті. З дні тому їв копчену свинину домашнього приготування, пив алкоголь. При огляді: у свідомості, температура тіла - 36,2оС, зіниці розширені, реакція на світло млява, птоз, горизонтальний ністагм. Блювотний і глотковий рефлекси не викликаються. Менінгеальні ознаки негативні. Який найбільш імовірний діагноз? The patient came to the clinic on the 2nd day of the illness with complaints of weakness, double vision, dry mouth. Since the previous day, he had been eating homemade smoked pork, drinking alcohol . On examination: conscious, body temperature - 36.2oC, pupils dilated, reaction to light sluggish, ptosis, horizontal nystagmus. Vomiting and pharyngeal reflexes are not elicited. Meningeal signs are negative. What is the most likely diagnosis?

Отруєння невідомою речовиною Poisoning by an unknown substance

Ішемічний інсульт мозку Ischemic brain stroke

Отруєння алкоголем Alcohol poisoning

Енцефаліт Encephalitis

Ботулізм Botulism

35 / 130
Породілля 27-ми років на 10 добу після пологів звернулася в жіночу консультацію зі скаргами на періодичний переймоподібний біль внизу живота. Пологи перші, термінові, нормальні. При обстеженні температура тіла - 36,8°С’, пульс -68/хв., AT- 115/75 мм рт.ст. Грудні залози помірно нагрублі, соски чисті. Живіт м’який, безболісний. Дно матки на рівні лона. Лохії помірні серозно-кров’янисті. Який можна поставити діагноз? On the 10th day after giving birth, a 27-year-old woman in labor went to the women's consultation with complaints of periodic spasm-like pain in the lower abdomen. The first delivery, urgent, normal. During the examination, the temperature body temperature - 36.8°С', pulse -68/min., AT- 115/75 mm Hg. Breasts are moderately thickened, nipples are clean. Abdomen is soft, painless. The bottom of the uterus is at the level of the womb. Lochia is moderate serous - bloody. What can be diagnosed?

Нормальний перебіг післяпологового періоду Normal course of the postpartum period

Лактостаз Lactostasis

Субінволюція матки Subinvolution of uterus

Післяпологовий ендометрит Postpartum endometritis

Післяпологовий метроендометрит Postpartum metroendometritis

36 / 130
Жінка 36-ти років скаржиться на біль, обмеження рухів у дрібних суглобах рук, утруднене ковтання твердої їжі, слабкість, сухий кашель. Об’єктивно: шкіра кистей та передпліччя щільна, гладенька. Проксимальні суглоби II-IV пальців кистей рук набряклі, болючі при пальпації. Над легенями сухі розсіяні хрипи, межі серця зміщені вліво на 2 см, тони приглушені. У крові: швидкість осідання еритроцитів - 36 мм/год, 7-глобуліни - 24%. У сечі: змін немає. Який найбільш імовірний діагноз? A 36-year-old woman complains of pain, limitation of movement in the small joints of the hands, difficulty swallowing solid food, weakness, dry cough. Objectively: the skin of the hands and the forearm is dense, smooth. Proximal joints II-IV of the fingers of the hands are swollen, painful on palpation. Over the lungs, dry scattered rales, the borders of the heart are shifted to the left by 2 cm, tones are muffled. In the blood: erythrocyte sedimentation rate - 36 mm/h, 7- globulins - 24%. In the urine: there are no changes. What is the most likely diagnosis?

Системний червоний вовчак Systemic lupus erythematosus

Системна склеродермія Systemic scleroderma

Саркоїдоз Sarcoidosis

Дерматоміозит Dermatomyositis

Ревматоїдний артрит Rheumatoid arthritis

37 / 130
Впродовж 3-ї доби життя шкіра хлопчика набула жовтушного кольору. При народженні вага 3550 г, довжина 53 см. Стан дитини задовільний, фізичний розвиток пропорційний, активно смокче. Живіт м’який, печінка виступає з-під реберної дуги на 1,5-2,0 см, еластична, селезінка не пальпується. Стілець меконіальний, сеча світла, прозора. Найбільш імовірний діагноз: During the 3rd day of life, the boy's skin acquired a yellowish color. At birth, weight 3550 g, length 53 cm. The child's condition is satisfactory, physical development is proportional, he actively sucks. The abdomen is soft, the liver protrudes from under the costal arch by 1.5-2.0 cm, it is elastic, the spleen is not palpable. The stool is meconial, the urine is light, transparent. The most likely diagnosis:

Анемія Мінковського-Шоффара Minkowski-Shofar Anemia

Фізіологічна жовтяниця Physiological jaundice

Гемолітична хвороба новонароджених Hemolytic disease of the newborn

Внутрішньопечінкова гіпоплазія Intrahepatic hypoplasia

Фетальний гігантоклітинний гепатит Fetal giant cell hepatitis

38 / 130
Хвора 34-х років надійшла до гінекологічного відділення зі скаргами на рясні кров’яні виділення зі статевих шляхів протягом 3 тижнів, що розпочались у термін чергової менструації. В анамнезі 2 фізіологічних пологів; гінекологічні захворювання заперечує. При огляді в дзеркалах слизова піхви та шийки матки чистВиділення кров’яні, рясні. При бімануальному дослідженні патології не виявлено. Діагноз: A 34-year-old patient came to the gynecological department with complaints of profuse bleeding from the genital tract for 3 weeks, which began at the time of the next menstruation. In the anamnesis 2 physiological births; denies gynecological diseases. When examined in mirrors, the mucous membrane of the vagina and cervix is clean. Discharges are bloody, abundant. Bimanual examination does not reveal pathology. Diagnosis:

Кровоточива ерозія шийки матки Bleeding erosion of the cervix

Аномальна маткова кровотеча Abnormal uterine bleeding

Аборт у ходу Abort in progress

Альгодисменорея Algodysmenorrhea

Рак ендометрію Endometrial cancer

39 / 130
При проведенні поточного профілактичного медичного огляду робітника цеху з виробництва автомобільних акумуляторів встановлено: шкіра бліда з сірувато-землистим відтінком, видимі слизові бліді. На яснах, переважно біля передніх зубів, є зміна кольору слизової у бузковий колір у вигляді смуги. Є тремор пальців рук. При пальпації рук є біль по ходу нервів. Назвіть токсичну сполуку та основний шлях її надходження до організму людини в умовах виробництва: During the current preventive medical examination of a worker in the workshop for the production of automobile batteries, it was established: the skin is pale with a grayish-earthy hue, the mucous membranes are pale. On the gums, mainly near the front teeth , there is a change in the color of the mucous membrane to a lilac color in the form of a band. There is a tremor of the fingers. When palpating the hands, there is pain along the course of the nerves. Name the toxic compound and the main route of its entry into the human body under production conditions:

Арсен, інгаляційний Arsenic, inhalation

Свинець, інгаляційний Lead, inhalation

Ртуть, пероральний Mercury, oral

Кадмій, перкутанний Cadmium, percutaneous

Нікель, перкутанний Nickel, Percutaneous

40 / 130
Хворий скаржиться на гнійні виділення з пенісу вранці перед сечовиділенням, різі, полакіурію. Визначена піурія при проведені трьохстаканної проби в першій порції сечі. Поставте діагноз: The patient complains of purulent discharge from the penis in the morning before urinating, cuts, pollakiuria. Pyuria was determined when a three-glass sample was taken in the first portion of urine. Make a diagnosis:

Хронічний простатит Chronic prostatitis

Гострии цистит Acute cystitis

Гострий балапопостит Acute balapoposthitis

Гострий уретрит Acute urethritis

Хронічний пієлонефрит Chronic pyelonephritis

41 / 130
На радіаційно забрудненій території продукти харчування містять радіонукліди: цезій (137 Cs) та стронцій (90 Sr). З метою зменшення всмоктування радіонуклідів та підвищення радіорезистентності організму людей, які вживають ці продукти, рекомендується: In a radiation-contaminated territory, food products contain radionuclides: cesium (137 Cs) and strontium (90 Sr). In order to reduce the absorption of radionuclides and increase the radioresistance of the body of people who use these products, it is recommended:

Збагачення раціону харчування пектинами та специфічними антагоністами цезію і стронцію (калій, кальцій) Enrichment of diet with pectins and specific antagonists of cesium and strontium (potassium, calcium)

Збагачення раціону харчування мікроелементами Enrichment of diet with trace elements

Дотримання режиму харчування Adherence to the diet

Збагачення раціону харчування вітамінами Enrichment of diet with vitamins

Збагачення раціону харчування повноцінними білками Enrichment of diet with complete proteins

42 / 130
Анестезіолог проводить наркоз під час ургентної операції з приводу розлитого перитоніту жінці 65-ти років вагою 135 кг. Спроби інтубації трахеї після введення міорелаксантів тричі поспіль виявились невдалими. Штучна вентиляція легень через лицьову маску ефективна. Яким чином забезпечити прохідність дихальних шляхів і проведення штучної вентиляції легень? The anesthesiologist administers anesthesia during an emergency operation for diffuse peritonitis to a 65-year-old woman weighing 135 kg. Attempts to intubate the trachea after the administration of muscle relaxants were unsuccessful three times in a row. Artificial ventilation lungs through a face mask is effective. How to ensure patency of the respiratory tract and artificial ventilation of the lungs?

Проводити штучну вентиляцію легень через лицьову маску Perform artificial lung ventilation through a face mask

Проводити штучну вентиляцію легень, встановивши орофарингеальний повітровід Perform artificial lung ventilation by installing an oropharyngeal airway

Проводити штучну вентиляцію легень, встановивши назофарингеальний повітровід Carry out artificial lung ventilation by installing a nasopharyngeal airway

Проводити штучну вентиляцію легень, встановивши ларингеальну маску Carry out artificial lung ventilation by installing a laryngeal mask

Проводити штучну вентиляцію легень через назальні канюлі Perform artificial lung ventilation through nasal cannulas

43 / 130
Хворий П., 36 років скаржиться на постійний тупий біль у лівому підребер'ї, після прийому жирної та копченої їжі, блювоту, яка не приносить полегшення. Калові маси блискучі, с неприємним запахом. Хворіє протягом 8 років; зловживає алкоголем, багато курить. Об'єктивно: знижененого харчування. Шкіра бліда і суха. Язик з білим нашаруванням. Живіт помірно здутий, відзначається біль в зоні Шоффара, Губергриця-Скульського, точках Дежардена, Мейо-Робсона. Про яке захворювання можна подумати в першу чергу? Patient P., 36 years old, complains of constant dull pain in the left hypochondrium, after eating fatty and smoked food, vomiting that does not bring relief. Fecal masses shiny, with an unpleasant smell. Ill for 8 years; abuses alcohol, smokes a lot. Objectively: poor nutrition. Skin is pale and dry. Tongue with white coating. Abdomen is moderately distended, pain is noted in the area of Shofar, Gubergryts-Skulsky, Desjardins points , Mayo-Robson. What disease can you think of first?

Виразкова хвороба Ulcer disease

Хронічний панкреатит Chronic pancreatitis

Хронічний ентероколіт Chronic enterocolitis

Хронічний холецистит Chronic cholecystitis

Хронічний гастродуоденіт Chronic gastroduodenitis

44 / 130
До лікаря жіночої консультації звернулася жінка, 6-7 тижнів вагітності. З анамнезу з’ясовано, що з 15-ти років хворіє на цукровий діабет у тяжкій формі. За заключенням окуліста ретинопатія II ст. Мала 2 вагітності, які закінчилися мертвонародженням. Вміст цукру в крові -15 ммоль/л. При бімануальному дослідженні встановлено, що матка збільшена (як при вагітності, терміном до 7 тижнів). Яка тактика лікаря? A woman, 6-7 weeks pregnant, turned to a doctor for a women's consultation. From the anamnesis, it was found that she has been suffering from diabetes in a severe form since she was 15 years old. According to the oculist's conclusion, retinopathy of the II stage. She had 2 pregnancies that ended in stillbirth. The blood sugar content is -15 mmol/l. During a bimanual examination, it was found that the uterus is enlarged (as in pregnancy, up to 7 weeks). What are the doctor's tactics?

Показано переривання вагітності Pregnancy termination is shown

Консультація ендокринолога Endocrinologist consultation

Госпіталізація в критичні терміни вагітності Hospitalization during critical periods of pregnancy

Пролонгування вагітності під контролем рівня цукру в крові Prolongation of pregnancy under blood sugar control

Консультація генетика Genetic consultation

45 / 130
Пацiєнтка 20-ти рокiв проходить лiкування з приводу анемiї (гемоглобiн - 72 г/л). Пiвтора роки тому пiсля мимовiльного викидня у термiнi 16 тижнiв та крововтрати, вiдмiчає зниження пам’ятi, втомлюванiсть, втрату апетиту, сухiсть шкiри, ламкiсть нiгтiв, набряклiсть, порушення менструальної функцiї. Об’єктивно: артерiальний тиск - 80/55 мм рт.ст., пульс - 54/хв., зрiст - 168 см, вага - 48 кг, гiпоплазiя статевих органiв. Призначення якого з перерахованих препаратiв буде найбiльш доречним цiй пацiєнтцi? A 20-year-old female patient is being treated for anemia (hemoglobin - 72 g/l). One and a half years ago, after a spontaneous miscarriage at 16 weeks and blood loss, she memory loss, fatigue, loss of appetite, dry skin, brittle nails, swelling, menstrual dysfunction Objectively: blood pressure - 80/55 mm Hg, pulse - 54/min, height - 168 cm, weight - 48 kg, genital hypoplasia. Which of the listed drugs would be most appropriate for this patient?

Метотрексат Methotrexate

Iмуноглобулiн людини нормальний Human immunoglobulin is normal

Гiдроксихлорохiн Hydroxychloroquine

Гiдрокортизон Hydrocortisone

Iнфлiксимаб Infliximab

46 / 130
Лікар швидкої допомоги під час надання медичної допомоги особі, витягнутої із зашморгу родичами, відзначив: відсутність пульсу на сонних артеріях, відсутність свідомості, самостійного дихання, корнеальних рефлексів та наявність трупних плям на спині і задній поверхні кінцівок. За якими ознаками можна констатувати настання смерті? The emergency physician, while providing medical assistance to a person pulled out of a coma by relatives, noted: the absence of a pulse on the carotid arteries, the absence of consciousness, independent breathing, corneal reflexes and the presence corpse spots on the back and back surface of the limbs. By what signs can one ascertain the onset of death?

Відсутність самостійного дихання Lack of spontaneous breathing

Відсутність корнеальних рефлексів Absence of corneal reflexes

Наявність трупних плям Presence of corpse stains

Відсутність свідомості Lack of consciousness

Відсутність пульсу No pulse

47 / 130
Хворий госпіталізований у відділення без свідомості. Хворіє на цукровий діабет упродовж 5 років. Отримує інсулін пролонгованої дії у дозі 24 ОД вранці та 18 ОД ввечері. Раптово втратив свідомість. Шкіра волога, тонус м’язів кінцівок підвищений. Тонус очних яблук нормальний. Пульс – 96/хв, АТ - 120/80 мм рт. ст. Тони серця звичайної звучності. Дихання ритмічне. Язик вологий. Менінгеальні симптоми відсутні. Яка терапія необхідна в першу чергу? The patient is hospitalized in the unconscious department. He has been suffering from diabetes for 5 years. He receives long-acting insulin at a dose of 24 units in the morning and 18 units in the evening. He suddenly lost consciousness. The skin is moist, the tone of the muscles of the limbs is increased. The tone of the eyeballs is normal. Pulse - 96/min, blood pressure - 120/80 mm Hg. Heart sounds of normal sound. Breathing is rhythmic. The tongue is moist. There are no meningeal symptoms. What therapy is necessary in first of all?

Введення інсуліну короткої дії внурішньовенно. Introduction of short-acting insulin intravenously.

Введення 40% розчину глюкози внутрішньовенно. Introduction of 40% glucose solution intravenously.

Введення адреналіну підшкірно. Injection of epinephrine subcutaneously.

Введення 4% розчину натрію гідрокарбонату внутрішньовенно. Introduction of 4% sodium bicarbonate solution intravenously.

Введення гідрокортизону внутрішньовенно. Introduction of hydrocortisone intravenously.

48 / 130
Хвора 25-ти років скаржиться на наявність декількох щільних, болючих вузлів у правій пахвовій ділянці, підвищення температури тіла до 38°С, загальну слабкість. Хворіє 3 доби. Локально: в правій пахвовій області є три вузлоподібних утворення, які підвищуються над поверхнею шкіри, розмірами від 0,5 см до 1,5 см в діаметрі, різко болючі при пальпації, з чіткими контурами, шкіра над ними багрово-синюшного відтінку, підняття верхньої кінцівки обмежено через біль. Поставте діагноз: A 25-year-old patient complains of the presence of several dense, painful nodes in the right armpit, an increase in body temperature to 38°C, general weakness. She has been ill for 3 days. Locally: in the right axillary region there are three nodular formations that rise above the surface of the skin, the size of which is from 0.5 cm to 1.5 cm in diameter, sharply painful when palpated, with clear contours, the skin above them has a purplish-bluish tint, raising the upper limbs are limited due to pain. Diagnose:

Гнійний лімфаденіт Suppurative lymphadenitis

Абсцес Abscess

Гнійний гідраденіт Suppurative hidradenitis

Фурункул Furnish

Карбункул Carbuncle

49 / 130
Хворий 41-го року доставлений в торакальне відділення з ножовим пораненням грудної клітки зліва. При огляді: загальний стан вкрай важкий. Шкіра та видимі слизові синюшні. Дихання важке, уривчасте, ЧД- 32/хв. При перкусії грудної клітки межі серця розширені, Пульс - 120/хв., AT- 60/20 мм рт.ст, Яка тактика ведення хворого? A 41-year-old patient was brought to the thoracic department with a stab wound to the chest on the left side. On examination: the general condition is extremely serious. The skin and visible mucous membranes are bluish. Breathing is difficult, intermittent, heart rate - 32/min. When percussing the chest, the borders of the heart are expanded, Pulse - 120/min, AT - 60/20 mm Hg, What are the tactics of managing the patient?

Невідкладна торакотомія Urgent thoracotomy

Горакоцентез Horacocentesis

Рентгенографія ОГК Roentgenography of OGK

Плевральна пункція Pleural puncture

Торакоскопія Thoracoscopy

50 / 130
У відділення доставлено пацієнта у важкому стані; температура - Зб,5оС загальмований, риси обличчя загострені, язик сухий, шкірна складка не розправляється. Тахіпноє до 28/хв., пульс - 126/хв.. AT 60/40 мм рт.ст, живіт безболісний. При госпіталізації блювота фонтаном, водянисті випорожнення кишечника, що нагадували рисовий відвар. З епіданамнезу: тиждень тому повернувся з Судану (Африка). Оберіть стартову терапію для невідкладної допомоги: A patient was brought to the department in a serious condition; temperature - Zb.5oC, retarded, facial features are sharpened, tongue is dry, skin fold does not straighten. Tachypnea up to 28/min. , pulse - 126/min.. AT 60/40 mm Hg, the abdomen is painless. During hospitalization, vomiting with a fountain, watery bowel movements that resembled rice broth. From the epidanamnesis: returned from Sudan (Africa) a week ago. Choose the initial therapy for emergency care:

Введення колоїдних розчинів Introduction of colloidal solutions

Введення свіжозамороженної плазми Introduction of fresh frozen plasma

Введення осмодіуретиків Introduction of osmodiuretics

Введення кристалоїдних розчинів Introduction of crystalloid solutions

Введення антибіотиків Introduction of antibiotics

51 / 130
Х-ру К, 58р., турбують часті запаморочення, кволість, схудла на 14 кг за 2 місяці. В анамнезі – перенесений туберкульоз. Об-но: посилена пігментація шкіри, дефіцит маси тіла – 10 кг, АТ 90/55 мм рт. ст. В крові: Na –117 ммоль/л, К – 6,4 ммоль/л, Cl – 116 ммоль/л, [НСО3-] – 27 мекв/л, глюкоза - 8,9 ммоль/л. Ваш діагноз? Kh-ru K, 58 years old, worried about frequent dizziness, weakness, lost 14 kg in 2 months. History of tuberculosis. Ob-no: increased skin pigmentation, body weight deficit – 10 kg, blood pressure 90/55 mm Hg In blood: Na – 117 mmol/l, K – 6.4 mmol/l, Cl – 116 mmol/l, [HСО3-] – 27 meq/l, glucose - 8.9 mmol/l. Your diagnosis?

Тромбоз ниркових вен Renal vein thrombosis

Наднирникова недостатність Adrenal insufficiency

Пухлина, що секретує АКТГ ACTH-secreting tumor

Синдром Гудпасчера Goodpasture Syndrome

Хронічна ниркова недостатність Chronic renal failure

52 / 130
У жінки 34-х років з попередньо нормальною менструальною функцією цикли стали нерегулярними, за даними тестів функціональної діагностики - анову-ляторними. Молочні залози пальпаторно болячі, виділяється молоко (галакторея). Яке дослідження показано хворій в першу чергу? In a 34-year-old woman with previously normal menstrual function, the cycles became irregular, according to functional diagnostic tests - anovulatory. The mammary glands are painful on palpation, milk is secreted ( galactorrhea). What study is shown to the patient in the first place?

КТ головного мозку CT brain

Прогсстеронова проба Progsterone test

Ультразвукове дослідження органів малого тазу Ultrasound examination of pelvic organs

Визначення рівня пролактину Prolactin level determination

Визначення рівня гонадотропінів Determining the level of gonadotropins

53 / 130
При плановому обстеженні 22-річної вагітної (ЗО тижнів) двічі у сечі виявлена ізольована бактеріурія. Вагітність перебігає без патології. Визначте тактику ведення вагітної: During a routine examination of a 22-year-old pregnant woman (3 weeks), isolated bacteriuria was detected twice in the urine. The pregnancy proceeds without pathology. Determine the tactics of managing the pregnant woman:

Фізіотерапевтичне лікування Physiotherapy treatment

Призначення ампіциліну Prescription of ampicillin

Динамічне спостереження Dynamic Observation

Фітотерапія Phytotherapy

Призначення ципрофлоксацину Ciprofloxacin Prescription

54 / 130
У 10-річної дівчинки на тлі підвищеної до 37,5С температури тіла спостерігаються помірне збільшення лімфовузлів шиї, сірувато-білі нашарування на мигдаликах у вигляді щільної плівки. Плівка важко знімається, поверхня під нею кро виточить. Набряку шиї немає. Попереднім діагноз: A 10-year-old girl, against the background of a body temperature raised to 37.5C, has a moderate increase in neck lymph nodes, grayish-white layering on the tonsils in the form of a dense film. The film is difficult is removed, the surface under it will bleed. There is no swelling of the neck. Preliminary diagnosis:

Стерптококовий тонзилофарингіт Streptococcal tonsillopharyngitis

Ангіна Симановського-Венсана Symanovsky-Vincent angina

Інфекційний мононуклеоз Infectious mononucleosis

Скарлатина Scarlatina

Дифтерія мигдаликів Diphtheria of tonsils

55 / 130
Чоловік 38-ми років скаржиться на безсоння, пригнічений настрій, періодичну тривогу, жахливі сновидіння, в яких ’’бачить смерть своїх побратимів’’. Два місяці тому повернувся з зони бойових дій. Який імовірний діагноз? A 38-year-old man complains of insomnia, depressed mood, periodic anxiety, terrible dreams in which he 'sees the death of his brothers'. Two months ago he returned from the combat zone. What is the probable diagnosis?

Біполярний афективний розлад Bipolar affective disorder

Посттравматичний стресовий розлад Post-traumatic stress disorder

Депресивний розлад Depressive disorder

Шизофренія Schizophrenia

Обсесивно-компульсивний розлад Obsessive-compulsive disorder

56 / 130
У пацієнта протягом тижня лихоманка, що супроводжується ознобами й вираженою пітливістю. Температура підвищується у вечірні години через день. Два тижні тому повернувся з Ефіопії. Яке лабораторне дослідження необхідно провести в першу чергу? The patient has had a fever for a week, accompanied by chills and severe sweating. The temperature rises in the evening every other day. He returned from Ethiopia two weeks ago. What laboratory test should be performed first of all?

Рентгенографію органів грудної клітки X-ray of chest organs

Бактеріологічний посів калу Bacterial culture of feces

Копроцитограму Coprocytogram

Мікроскопія мазка й товстої краплі крові Microscopy of blood smear and thick drop

Загальний аналіз сечі General urinalysis

57 / 130
У хворої 34-х років сильний біль глибоко в орбіті, головний біль, підвищення температури тіла. Захворювання виникло гостро. Об’єктивно: повіки набряклі, шкіра червоного кольору, екзофтальм, обмеження рухів очного яблукВаш діагноз: A 34-year-old patient has severe pain deep in the orbit, headache, increased body temperature. The disease occurred acutely. Objectively: the eyelids are swollen, the skin is red , exophthalmos, restriction of eyeball movements Your diagnosis:

Абсцес повіки Eyelid abscess

Виразка рогівки Corneal ulcer

Ретробульбарний неврит Retrobulbar neuritis

Флегмона орбіти Phlegmon of the orbit

Блефарит Blepharitis

58 / 130
До гінеколога звернулася жінка 36-ти років зі скаргами на нерегулярні мізерні менструації впродовж останніх 6 місяців, посилений ріст волосся на обличчі та ’’огрубіння” голосу. В анамнезі два викидні, одні передчасні пологи. За даними УЗД матка та яєчники без особливостей. Яке обстеження доцільно провести для встановлення причини гормональних порушень? A 36-year-old woman came to the gynecologist with complaints of irregular scanty menstruation for the past 6 months, increased growth of facial hair and 'roughening' of the voice. In the anamnesis two miscarriages, one premature birth. According to the ultrasound of the uterus and ovaries, there are no abnormalities. What examination should be carried out to determine the cause of hormonal disorders?

Проведення гістероскопи Hysteroscopy

Гормональне обстеження Hormonal examination

Взяття біопсії ендометрію Endometrial biopsy

Проведення рентген-дослідження турецького сідла Carrying out an X-ray examination of the Turkish saddle

Каріотипування Karyotyping

59 / 130
Дівчинка 4-х років захворіла гостро після перенесеної 3 тижні тому стрептококової інфекції на шкірі. З’явилися набряки обличчя, нижніх кінцівок, головний біль, темний колір сечі (колір ’’кока-коли”). AT- 125/60 мм рт.ст. Аналіз сечі: білок - 3,3°/ОО, питома вага - 1012, лейкоцити - 1-3 в п/з, еритроцити вкривають все п/з. Добова протеїнурія - 980 мг. Діурез - 550 мл. Загальний білок крові - 60 г/л. Альбуміни - 55%. Холестерин - 4,5 ммоль/л. Креатинін сироватки - 89 мкмоль/л. Який найбільш імовірний діагноз? A 4-year-old girl became acutely ill after suffering a streptococcal infection on the skin 3 weeks ago. Swelling of the face, lower limbs, headache, dark urine color appeared ( color 'Coca-Cola'). AT- 125/60 mm Hg. Urine analysis: protein - 3.3°/OO, specific gravity - 1012, leukocytes - 1-3 in p/z, erythrocytes cover all p /z. Daily proteinuria - 980 mg. Diuresis - 550 ml. Total blood protein - 60 g/l. Albumin - 55%. Cholesterol - 4.5 mmol/l. Serum creatinine - 89 μmol/l. What is the most likely diagnosis?

Гостре ураження нирок Acute kidney injury

Пієлонефрит Pyelonephritis

Сечокам’яна хвороба Urolithiasis

Гломерулонефрит, нефритичний синдром Glomerulonephritis, nephritic syndrome

Гломерулонефрит, нефротичний синдром Glomerulonephritis, nephrotic syndrome

60 / 130
В ході профогляду встановлено: у жінки 23-х років вагітностей не було. При бімануальному обстеженні виявлено: тіло матки нормальних розмірів, на передній стінці - щільне округле утворення на ніжці, пов’язане з маткою, розмірами в діаметрі до 6-ти см, неболюче, додатки без особливостей. За допомогою УЗД підтверджено діагноз субсерозної міоми матки. Який метод лікування слід запропонувати? During the professional examination, it was established that a 23-year-old woman had no pregnancies. Bimanual examination revealed: the body of the uterus is of normal size, on the front wall - a dense rounded formation on legs, connected to the uterus, up to 6 cm in diameter, painless, appendages without features. The diagnosis of subserous uterine myoma was confirmed by ultrasound. What method of treatment should be offered?

Екстирпація матки Uterus extirpation

Консервативна міомектомія Conservative myomectomy

Ампутація матки Uterine amputation

Дефундація матки Defundation of the uterus

Висока надпіхвова ампутація матки High supravaginal amputation of the uterus

61 / 130
Хворий 40-ка років скаржиться на біль в епігастральній ділянці натще, печію. При пальпації помірна болючість у пілородуоденальній зоні. Який метод діагностики буде найінформативнішим? A 40-year-old patient complains of pain in the epigastric region on an empty stomach, heartburn. On palpation, there is moderate tenderness in the pyloroduodenal area. What diagnostic method would be the most informative?

Внутрішньошлункова рН-метрія Intragastric pH-metry

Дихальний уреазний тест Breathing urease test

Фіброезофагогастродуоденоскопія Fibroesophagogastroduodenoscopy

УЗД черевної порожнини Ultrasound of the abdominal cavity

Рентгеноскопія шлунка та 12-палої кишки Rentoscopy of stomach and duodenum

62 / 130
Жінка 78-ми років, яка страждає на артеріальну гіпертензію, мала три епізоди раптової транзиторної втрати зору на ліве око. При аускультації сонних артерій в точках біфуркації з обох боків вислуховується шум. Яке додаткове дослідження найбільш доцільно призначити? A 78-year-old woman suffering from arterial hypertension had three episodes of sudden transient loss of vision in the left eye. On auscultation of the carotid arteries at the bifurcation points on both sides noise is heard. What additional research is most appropriate to prescribe?

Нейросонографія Neurosonography

Люмбальна пункція Lumbar puncture

Дуплексне сканування судин шиї Duplex scan of neck vessels

МРТ головного мозку MRI brain

КТ головного мозку CT brain

63 / 130
17-річний хлопець на уроці фізичного виховання раптово втратив свідомість. На місці була проведена серцево-легенева реанімація. На момент приїзду швидкої свідомість до пацієнта повернулася. При зборі анамнезу стало відомо, що батько і дід дитини проходили обстеження та лікування у кардіолога, проте помирали в молодому віці. При фізикаль-ному огляді дитини встановлено наявність систолічного шуму в серці, гучність якого збільшується при зміні положення тіла. При ехокардіографічному дослідженні зареєстровано потовщення стінки лівого шлуночка більше 15 мм. Яке захворювання можна припустити у даного хворого? A 17-year-old boy suddenly lost consciousness during a physical education class. Cardiopulmonary resuscitation was performed on the spot. At the time of the arrival of the ambulance, the patient regained consciousness. During the collection of anamnesis it became known that the child's father and grandfather were examined and treated by a cardiologist, but died at a young age. Physical examination of the child revealed the presence of a systolic heart murmur, the volume of which increases when the body position changes. Echocardiographic examination revealed thickening of the left ventricular wall more than 15 mm. What disease can be assumed in this patient?

Неревматичний кардит Non-rheumatic carditis

Гіпертрофічна кардіоміопатія Hypertrophic cardiomyopathy

Вегетативна дисфункція Vegetative dysfunction

Дилатаційна кардіоміопатія Dilated cardiomyopathy

Гостра ревматична лихоманка Acute rheumatic fever

64 / 130
Хворий 33-х років лікується в відділенні інтенсивної терапії з приводу синдрому тривалого стиснення, гострої ниркової недостатності. При лабораторному обстеженні: сечовина - 27,14 ммоль/л; креатинін - 710 мкмоль/л; К - 73 ммоль/л; НЬ- 69 г/л; еритроцити - 2,4 • 1012 г/л. Яке ускладнення безпосередньо загрожує життю? A 33-year-old patient is being treated in the intensive care unit for long-term compression syndrome, acute renal failure. Laboratory examination: urea - 27.14 mmol/l; creatinine - 710 μmol/l; K - 73 mmol/l; Hb - 69 g/l; erythrocytes - 2.4 • 1012 g/l. What complication directly threatens life?

Респіраторний дистрес-синдром Respiratory distress syndrome

ДВЗ-синдром DVZ-syndrome

Гіперкаліємічна зупинка серця Hyperkalemic cardiac arrest

Гемічна гіпоксія Hemic hypoxia

Важка анемія Severe anemia

65 / 130
В ході обстеження дитини 5-ти років зі скаргами на постійний кашель з виділенням гнійного мокротиння та вологі хрипи справа у задньонижніх відділах виявлено: гнійний ендобронхіт ІІ-Ш ступеню, переважно справа, хлориди поту - 36 ммоль/л, на КТ - циліндричні бронхоектази в S9 та S10. Запропонуйте оптимальний метод лікування даної дитини: During the examination of a 5-year-old child with complaints of a constant cough with purulent sputum discharge and wet wheezing on the right in the lower back, it was found: purulent endobronchitis of the II-S degree, mainly on the right, sweat chlorides - 36 mmol/l, on CT - cylindrical bronchiectasis in S9 and S10. Suggest the optimal method of treatment for this child:

Видалення уражених сегментів легень Removal of affected lung segments

Лаваж бронхіального дерева Bronchial tree lavage

Генно-інженерна терапія Genetic engineering therapy

Тривала антибіотикотерапія Long-term antibiotic therapy

Кінезотерапія Kinesiotherapy

66 / 130
Хлопчик 10-ти років скаржиться на підвищення температури тіла до 38,5°С, припухлість суглобів пальців рук та ніг, ранкову скутість, біль у шийному відділі хребта. Проведено обстеження. У крові: еритроцити - 2,6 ¦ 1012/л, НЬ- 85 г/л, лейкоцити - 16, 5 • 109/л, е.- 2%, п,- 8%, с.-68%, л.- 16%, м,- 6%, швидкість осідання еритроцитів - 28 мм/год. Позитивний ревматоїдний фактор. Диспротеїнемія з гіперглобулінемією. Встановіть діагноз: A 10-year-old boy complains of an increase in body temperature to 38.5°C, swelling of the joints of fingers and toes, morning stiffness, pain in the cervical spine. An examination was carried out. In the blood: erythrocytes - 2.6 ¦ 1012/l, Hb - 85 g/l, leukocytes - 16.5 • 109/l, e.- 2%, p.- 8%, s.-68% , l.- 16%, m,- 6%, erythrocyte sedimentation rate - 28 mm/h. Positive rheumatoid factor. Dysproteinemia with hyperglobulinemia. Establish a diagnosis:

Реактивний артрит Reactive arthritis

Остеоартрит Osteoarthritis

Ювенільний ревматоїдний артрит Juvenile rheumatoid arthritis

Гостра ревматична лихоманка Acute rheumatic fever

Токсичний синовїїт Toxic synovitis

67 / 130
Хворий 30-ти років скаржиться на біль в ділянці серця, задишку, серцебиття при незначному навантаженні, набряки на нижніх кінцівках. Хворів на ентерові-русну інфекцію 3 тижні тому. Об’єктивно: ЧСС- 94/хв., AT- 100/70 мм рт.ст., межі серця розширені, послаблення 1 тону серця, систолічний шум над верхівкою. Титр АСЛ-О в нормі. ЕКГ: низький вольтаж QRS, подовження інтервалу Q-T. Який найбільш імовірний діагноз? A 30-year-old patient complains of pain in the heart area, shortness of breath, palpitations with slight exertion, swelling of the lower extremities. Patients with entero-cutaneous infection 3 weeks ago . Objectively: heart rate - 94/min., AT - 100/70 mm Hg, heart borders are expanded, weakening of 1 heart tone, systolic murmur over the apex. ASL-O titer is normal. ECG: low QRS voltage, QT prolongation What is the most likely diagnosis?

Ревматичний міокардит Rheumatic myocarditis

Інфекційний ендокардит Infective endocarditis

Ексудативний перикардит Exudative pericarditis

Стенокардія напруги Tension Angina

Вірусний міокардит Viral myocarditis

68 / 130
У дівчинки 11-ти років (в анамнезі часті ангіни) захворювання мало поступовий розвиток. З’явились дратівливість, неуважність, гіперкінези. При обстеженні виявлено порушення координації рухів, гіпотонію м’язів, гіперрефлексію. Для якого захворювання характерні такі зміни зі сторони центральної нервової системи? In an 11-year-old girl (with a history of frequent angina), the disease had a gradual development. Irritability, inattention, hyperkinesis appeared. During the examination, a violation of movement coordination was found, muscle hypotonia, hyperreflexia. What disease is characterized by such changes on the part of the central nervous system?

Гостра ревматична лихоманка Acute rheumatic fever

Системний червоний вовчак Systemic lupus erythematosus

Вегето-судинна дисфункція Vegeto-vascular dysfunction

Вузликовий поліартерїїт Polyarteritis nodosa

Системна склеродермія Systemic scleroderma

69 / 130
Хворий 65-ти років скаржиться на біль у правій нирковій ділянці, кров при сечовипусканні протягом усього акту, відсутність апетиту та слабкість. Об’єктивно: шкіра звичайного кольору, суха. При пальпації живота в правій нирковій ділянці - еластичне утворення, що зміщується, безболісне. Аналіз сечі - макрогематурія, атипові клітини. НЬ крові -110 г/л, ШОЕ- 45 мм/год. Яка найбільш імовірна патологія, що зумовлює таку картину? A 65-year-old patient complains of pain in the right renal area, blood during urination during the entire act, lack of appetite and weakness. Objectively: skin of normal color, dry. On palpation of the abdomen in the right renal area - an elastic mass that shifts, painless. Urinalysis - macrohematuria, atypical cells. Blood Hb - 110 g/l, ESR - 45 mm/h. What is the most likely pathology that causes such a picture ?

Пухлина правої нирки Right kidney tumor

Пухлина товстої кишки Colon tumor

Гострий пієлонефрит Acute pyelonephritis

Апендикулярний інфільтрат Appendicular infiltrate

Пухлина правого сечоводу Tumor of the right ureter

70 / 130
Чоловік 45-ти років доставлений у відділення невідкладної допомоги зі скаргами на раптовий інтенсивний біль у поперековій ділянці, часте болісне сечовиділення, блювання. Температура тіла - 36,8°С, пульс - 82/хв., артеріальний тиск -130/80 мм рт.ст. При фізикальному обстеженні болісність при пальпації поперекової ділянки відсутня, позитивний симптом Пастернацького. При лабораторному дослідженні у загальному аналізі сечі білок - 0,06 г/л, реакція - слабко кисла, лейкоцити - 3-4 в полі зору, еритроцити - 30-40 в полі зору. Який діагноз є найбільш імовірним? A 45-year-old man was brought to the emergency department with complaints of sudden intense pain in the lumbar region, frequent painful urination, vomiting. Body temperature - 36.8° C, pulse - 82/min., blood pressure -130/80 mm Hg. During physical examination, there is no pain during palpation of the lumbar region, positive Pasternacki's symptom. During laboratory research, in the general analysis of urine, protein - 0.06 g/l, reaction - weakly acidic, leukocytes - 3-4 in the field of vision, erythrocytes - 30-40 in the field of vision. What diagnosis is most likely?

Полікістоз нирок Polycystic kidney disease

Гостра ниркова недостатність Acute renal failure

Сечокам’яна хвороба Urolithiasis

Гострий гломерулонефрит Acute glomerulonephritis

Гострий пієлонефрит Acute pyelonephritis

71 / 130
Після ДТП у хлопчика 12-ти років з закритою травмою живота та переломом стегна зліва спостерігається чітко виражена загальмованість. Блідість шкіри, спад температури тіла. Артеріальний тиск - 75 мм рт.ст. Пульс - 120/хв., слабкого наповнення і напруги, нерівний. Подих прискорений, поверхневий. Визначте ступінь травматичного (гіповолемічного) шоку у потерпілого: After a road accident, a 12-year-old boy with a closed abdominal injury and a hip fracture on the left side has a clearly pronounced retardation. Skin pallor, a drop in body temperature. Blood pressure - 75 mm Hg. Pulse - 120/min., weak filling and tension, uneven. Breathing is accelerated, shallow. Determine the degree of traumatic (hypovolemic) shock in the victim:

Шок не визначається Shock not defined

Шок 4 ступеня (передагональний стан) Shock 4th degree (race state)

Шок 2 ступеня (середньої тяжкості) Shock 2 degree (moderate)

Шок 3 ступеня (тяжкий) Shock 3 degree (severe)

Шок 1 ступеня (легкий) Shock 1 degree (mild)

72 / 130
При огляді дитини віком 3 місяці, визначається асиметрія сідничних складок, обмеження відведення в правому кульшовому суглобі. На рентгенограмі кульшових суглобів - кути альфа справа 28 градусів, зліва - 32 градуси, ядра окостеніння відсутні. Вкажіть імовірний діагноз: When examining a 3-month-old child, the asymmetry of the gluteal folds, restriction of abduction in the right hip joint is determined. On the radiograph of the hip joints, alpha angles on the right are 28 degrees, on the left - 32 degrees, ossification nuclei are absent. Specify the probable diagnosis:

Вікова норма Age norm

Дисплазія лівого кульшового суглоба Left hip dysplasia

Рахіт Rickets

Двобічна дисплазія кульшових суглобів Bilateral hip dysplasia

Дисплазія правого кульшового суглоба Dysplasia of the right hip joint

73 / 130
Хвора 36-ти років скаржиться на задишку, відчуття стискання в правій половині грудної клітки, підвищення температури до 38,7°С, кашель з виділенням невеликої кількості слизово-гнійного харкотиння. Хворіє понад тиждень. Скарги пов’язує з переохолодженням. Об’єктивно: легкий акроціаноз губ, пульс ритмічний, 90/хв., артеріальний тиск -140/85 мм рт.ст. Права половина грудної клітки відстає в акті дихання. Перкусія - справа нижче кута лопатки прослуховується тупість з межею до верху. В цій ділянці дихання відсутнє. Який найбільш імовірний діагноз? A 36-year-old patient complains of shortness of breath, a feeling of tightness in the right half of the chest, an increase in temperature to 38.7°C, a cough with the release of a small amount of mucous purulent sputum. He has been sick for more than a week. He relates the complaints to hypothermia. Objectively: slight acrocyanosis of the lips, rhythmic pulse, 90/min., blood pressure -140/85 mm Hg. The right half of the chest lags behind in the act of breathing. Percussion - on the right below the angle of the scapula, dullness is heard with a border to the top. There is no breathing in this area. What is the most likely diagnosis?

Позагоспітальна пневмонія Community-acquired pneumonia

Абсцес легені Lung abscess

Ексудативний плеврит Exudative pleurisy

Ателектаз легені Atelectasis of the lung

ТЕЛА BODIES

74 / 130
Жінка 35-ти років звернулася до лікаря зі скаргами на зростаючий біль під час менструації протягом 1-го року. Тривалість менструального циклу 28 дні2 роки тому була проведена лапароскопічна перев’язка маткових труб. При пальпації матка збільшена до 8 тижнів вагітності, м’якої консистенції, болючПридатки з обох сторін не збільшені, при пальпації безболісні. Який діагноз є найбільш імовірним? A 35-year-old woman consulted a doctor with complaints of increasing pain during menstruation during the 1st year. The duration of the menstrual cycle is 28 days. 2 years ago, a laparoscopic surgery was performed 'tissue of fallopian tubes. On palpation, the uterus is enlarged up to 8 weeks of pregnancy, soft consistency, painful. The appendages on both sides are not enlarged, on palpation they are painless. What diagnosis is the most probable?

Гідросальпінкс Hydrosalpinx

Зовнішній ендометріоз External endometriosis

Аденоміоз Adenomyosis

Позаматкова вагітність Ectopic pregnancy

Полікістоз яєчників Polycystic ovary

75 / 130
Дівчинка 6-ти років надійшла зі скаргами на підвищення температури тіла до У1,2°С, часті та болісні сечовипускання, які з’явилися після переохолодження. У сечі: сеча каламутна, питома вага - 1012, білок - 0,033°Іоо, мікроскопія: лейкоцити - 40-45 в п/з, еритроцити - 8-9 в п/з (свіжі), епітелій плаский: 5-8 в п/з, слиз. Який етіологічний фактор у даному випадку найбільш імовірний? A 6-year-old girl came in with complaints of an increase in body temperature up to 1.2°С, frequent and painful urination, which appeared after hypothermia. In the urine : cloudy urine, specific gravity - 1012, protein - 0.033°Ioo, microscopy: leukocytes - 40-45 in p/z, erythrocytes - 8-9 in p/z (fresh), squamous epithelium: 5-8 in p/z , mucus. What is the most probable etiological factor in this case?

Escherichia coli Escherichia coli

Klebsiella pneumoniae Klebsiella pneumoniae

Proteus mirabilis Proteus mirabilis

Staphylococcus aureus Staphylococcus aureus

Candida albicans Candida albicans

76 / 130
Хворий 34-х років перебуває на лікуванні в психіатричній лікарні з приводу загострення шизофренії. Об’єктивно: перебуває в ліжку, рухливо загальмований, контакт відсутній. На запитання не відповідає. Поза одноманітна, пацієнт гі-помімічний, наявні симптом ’’хоботка’,’ воскова гнучкість м’язів, симптом ’’повітряної подушки’’. В такому стані лишається близько тижня. Харчування парентеральне. Визначте наявний синдром розладу рухово-вольової сфери: A 34-year-old patient is being treated in a psychiatric hospital due to an exacerbation of schizophrenia. Objectively: he is in bed, physically inhibited, there is no contact. When asked, no responds. The posture is monotonous, the patient is hypomimetic, the symptom of 'proboscis', waxy flexibility of the muscles, the symptom of 'air cushion' are present. He remains in this state for about a week. Nutrition is parenteral. Determine the syndrome of motor-volition disorder present :

Психогенний ступор Psychogenic stupor

Депресивний ступор Depressive stupor

Апатичний ступор Apathetic stupor

Екзогенний ступор Exogenous stupor

Кататонічний ступор Catatonic stupor

77 / 130
З дому надійшла дитина 3-х років, у якої на тлі гіпертермічного синдрому, спричиненого грипом, протягом 50-ти хвилин тривають тоніко-клонічні генера-лізовані судоми. Терапія не проводилася. Для негайного лікування судомного синдрому необхідно: A 3-year-old child came from home, who has been having tonic-clonic generalized convulsions for 50 minutes against the background of hyperthermic syndrome caused by the flu. Therapy was not carried out. For the immediate treatment of a convulsive syndrome, it is necessary:

Ввести лоразепам внутрішньовенно Inject lorazepam intravenously

Ввести кальцію глюконат внутрішньовенно Enter calcium gluconate intravenously

Терміново накласти дитині холодні компреси Urgently apply cold compresses to the child

Призначити парацетамол у високих дозах Prescribe paracetamol in high doses

Ввести фенобарбітал внутрішньом’язово Inject phenobarbital intramuscularly

78 / 130
У хворого 46-ти років після перенесеної пневмонії скарги на біль в лівій половині грудної клітки, підвищення температури тіла до 39°С, непродуктивний кашель, загальну слабкість. Хворіє два тижні. При аускультації в нижніх відділах зліва дихання не вислуховується, при перкусії - притуплення легеневого звуку. Який додатковий інструментальний метод дослідження є найбільш інформативним для встановлення діагнозу? A 46-year-old patient complains of pain in the left half of the chest, an increase in body temperature to 39°C, a non-productive cough, general weakness after suffering pneumonia. The patient two weeks. During auscultation in the lower parts on the left, breathing is not heard, during percussion - dulling of the lung sound. What additional instrumental research method is the most informative for establishing a diagnosis?

Бронхографія Bronchography

Комп’ютерна томографія Computed tomography

Спірографія Spirography

Бронхоскопія Bronchoscopy

Медіастиноскопія Mediastinoscopy

79 / 130
Пацієнт скаржиться на виражений біль в анальному каналі та періанальній ділянці. При огляді періанальної ділянки на 3, 7 і 11 годин умовного циферблата виявляються пухлиноподібні утворення, всередині яких визначаються тромби; помірно виражена періанальна гіперемія. Ваш попередній діагноз: The patient complains of severe pain in the anal canal and perianal area. When examining the perianal area at 3, 7 and 11 o'clock of the conventional dial, tumor-like formations are revealed, inside which blood clots are identified ; moderately pronounced perianal hyperemia. Your previous diagnosis:

Хронічна анальна тріщина Chronic anal fissure

Хронічний геморой Chronic hemorrhoids

Гострий парапроктит Acute paraproctitis

Гострий геморой Acute hemorrhoids

Випадання прямої кишки Rectal prolapse

80 / 130
Хворому 65-ти років з приводу застійної серцевої недостатності призначена фармакотерапія (фуросемід, спіронола-ктон, бісопролол, раміприл). Через 2 місяці звернувся до лікаря з приводу розвитку гінекомастїї. Запідозрено побічну дію спіронолактону На який з наведених препаратів можна його замінити у схемі лікування цього хворого? A 65-year-old patient was prescribed pharmacotherapy (furosemide, spironolactone, bisoprolol, ramipril) for congestive heart failure. After 2 months, he consulted a doctor about the development gynecomastia. A side effect of spironolactone is suspected. Which of the following drugs can replace it in the treatment regimen of this patient?

Ніфедипін Nifedipine

Торасемід Torasemide

Івабрадин Ivabradin

Еплеренон Eplerenone

Триметазидин Trimetazidine

81 / 130
У заводській їдальні виникло харчове отруєння, клініка якого вказувала на стафілококову етіологію. Захворіло 15 чоловік. Які матеріали від хворого необхідно надіслати на дослідження у лабораторію, щоб підтвердити харчове отруєння? Food poisoning occurred in the factory canteen, the clinic of which indicated a staphylococcal etiology. 15 people fell ill. What materials from the patient should be sent to the laboratory for research to confirm food poisoning ?

Кров (клінічний аналіз) Blood (clinical analysis)

Сечу Urine

Блювотні маси хворого Vomiting masses of the patient

Слину Saliva

Кров на гемокультури Blood for blood cultures

82 / 130
На прийом до педіатра звернулась мати із 4-місячною дитиною зі скаргами на підвищення температури до 37,5°С. Дитина народжена доношеною, 8-9 балів за шкалою Ангар. При аналізі медичної документації виявлено, що у дитини немає щеплення проти туберкульозу. В які часові терміни повинно було бути проведене щеплення проти туберкульозу? A mother with a 4-month-old child came to the pediatrician with complaints of an increase in temperature to 37.5°C. The child was born full-term, 8-9 points on the scale Hangar. During the analysis of the medical documentation, it was found that the child has not been vaccinated against tuberculosis. When should the vaccination against tuberculosis have been carried out?

Протягом першого тижня життя During the first week of life

Протягом перебування в пологовому будинку після попередньої проби Манту During the stay in the maternity hospital after the previous Mantoux test

Доношена дитина, 3-5 доба життя Full-term baby, 3-5 days old

Протягом перших 24 годин життя During the first 24 hours of life

Доношена дитина, не пізніше 48 годин від народження Full-term baby, no later than 48 hours after birth

83 / 130
Пацієнтка 28-ми років звернулась зі скаргами на нудоту, слабкість, зниження працездатності. Остання менструація була 2 місяці тому. Заміжня. Об’єктивне обстеження: загальний стан задовільний. Пульс - 74/хв., ритмічний. AT- 110/70 мм рт.ст. При піхвовому дослідженні: зовнішні статеві органи розвинені правильно, ріст волосся - за жіночим типом. Піхва жінки, що не народжувала. Шийка матки не змінена, ціанотична, конічної форми, зовнішнє вічко закрите. Матка рухома, безболісна, збільшена до 9-10 тижнів вагітності. Придатки матки без особливостей. Склепіння глибокі, вільні. Виділення слизові, світлі. Який передбачуваний діагноз? A 28-year-old female patient complained of nausea, weakness, reduced work capacity. The last menstruation was 2 months ago. She is married. Objective examination: the general condition is satisfactory . Pulse - 74/min., rhythmic. AT- 110/70 mm Hg. During vaginal examination: external genitalia are developed correctly, hair growth is according to the female type. Vagina of a woman who has not given birth. Cervix is not changed, cyanotic , conical in shape, the outer eye is closed. The uterus is mobile, painless, enlarged up to 9-10 weeks of pregnancy. Uterine appendages without features. The vaults are deep, free. The discharge is mucous, light. What is the expected diagnosis?

Лейоміома матки Leiomyoma of the uterus

Рак тіла матки Cancer of the uterine body

Вагітність І, 9-10 тижнів Pregnancy I, 9-10 weeks

Позаматкова вагітність Ectopic pregnancy

Міхуровий замет Bubble blizzard

84 / 130
В робочій зоні коваля визначені високі рівні показників вібрації та висока концентрація чадного газу. Яка дія чинників виробничого середовища на організм робітників? In the blacksmith's working area, high levels of vibration indicators and a high concentration of carbon monoxide are determined. What is the effect of the factors of the production environment on the body of workers?

Комплексна Comprehensive

Поєднана Combined

Специфічна Specific

Комбінована Combined

Роздільна Separate

85 / 130
У хворого 47-ми років на восьмий день після операції з приводу панкреонекрозу, перитоніту з’явилось підвищення температури тіла до 39°С, анемія, лейкоцитоз, гіпопротеїнемія, спленомегалія, токсична енцефалопатія. Яке ускладнення має розвиток у цій ситуації? On the eighth day after surgery for pancreatic necrosis, peritonitis, a 47-year-old patient developed an increase in body temperature to 39°C, anemia, leukocytosis, hypoproteinemia, splenomegaly, toxic encephalopathy. What complication does development have in this situation?

Внутрішньоочеревинна кровотеча Intraperitoneal bleeding

Загальна гнійна інфекція (сепсис) General purulent infection (sepsis)

Гостра ниркова недостатність Acute renal failure

Гострий менінгіт Acute meningitis

Гостра печінкова недостатність Acute liver failure

86 / 130
У породіллі через 3 тижні після пологів підвищилася температура тіла до 38°С, з’явилися остуда, слабкість та біль в лівій молочній залозі. Молочна залоза збільшена, нагрубла, болюча при пальпації; розм’якшення та флуктуації в ділянці інфільтрату немає. В крові - помірний лейкоцитоз. Який найбільш імовірний діагноз? 3 weeks after giving birth, the mother's body temperature rose to 38°C, chills, weakness and pain in the left mammary gland appeared. The mammary gland is enlarged, rough , painful on palpation; there is no softening and fluctuation in the area of the infiltrate. In the blood - moderate leukocytosis. What is the most likely diagnosis?

Серозний мастит Serous mastitis

Лактостаз Lactostasis

Мастопатія Mastopathy

Абсцедивний мастит Abscessive mastitis

87 / 130
Жінка 45-ти років скаржиться на колькоподібний біль у правому підребер’ї, що виникає після вживання жирної їжі, фізичного навантаження, іррадіює в праву лопатку та праве плече, зменшується при прийомі спазмолітиків. Хворіє впродовж року, періодично зазначає пожовтіння склер під час нападів болю. Який метод обстеження слід призначити в першу чергу для встановлення діагнозу? A 45-year-old woman complains of colic pain in the right hypochondrium, which occurs after eating fatty food, physical exertion, radiates to the right shoulder blade and right shoulder, decreases when taking antispasmodics. He has been sick for a year, periodically notes yellowing of the sclera during pain attacks. What examination method should be prescribed first of all to establish a diagnosis?

Ультрасонографію Ultrasonography

Дуоденальне зондування 6- Wrong answer.Should not have been checked. Duodenal sounding 6- Wrong answer. Should not have been checked.

Рентгенографію органів черевної порожнини X-ray of abdominal organs

Фіброгастродуоденоскопію Fibrogastroduodenoscopy

Комп’ютерну томографію Computed tomography

88 / 130
Після прогулянки на вулиці в ясний сонячний день у дитини 8 місяців з проявами рахіту ГГ ступеню, підгострого перебігу, періоду розпалу, з’явилися судоми у вигляді карпопедального спазму. Вони свідчать про наявність у дитини: After a walk on the street on a clear sunny day, an 8-month-old child with manifestations of rickets of the HH degree, subacute course, in the flare-up period, developed convulsions in the form of carpopedal spasm. They indicate that the child has:

ГРВГ GRVH

Черепно-мозкової травми Traumatic brain injury

Епілепсії Epileptics

Спазмофілії Spasmophilia

Менінгіту Meningitis

89 / 130
Для вивчення залежності між тривалістю тютюнопаління та частотою виявлення злоякісних новоутворів легень використовується один із статистичних методів. Який саме метод статистичного аналізу доцільно використати в цьому випадку? One of the statistical methods is used to study the relationship between the duration of smoking and the frequency of detection of malignant lung neoplasms. Which statistical analysis method should be used in this case?

Комплексні оцінки здоров’я Comprehensive Health Assessments

Кореляційний аналіз Correlation Analysis

Динамічний аналіз Dynamic Analysis

Нормування інтенсивних показників Normalization of intensive indicators

Прямий метод стандартизації показників Direct method of standardization of indicators

90 / 130
Хворий звернувся до лікаря зі скаргами на збільшення шийних та пахвових лімфовузлів, важкість у лівому підребер’ї. При пальпації лімфовузли м’які, безболісні, не спаяні. Спленомегалія. У крові: лейкоцити - 70 Г/л, лімфоцити -80%, тіні Гумпрехта. Який імовірний діагноз? The patient turned to the doctor with complaints of an increase in cervical and axillary lymph nodes, heaviness in the left hypochondrium. Upon palpation, the lymph nodes are soft, painless, not fused. Splenomegaly . In the blood: leukocytes - 70 g/l, lymphocytes -80%, Humprecht's shadows. What is the probable diagnosis?

Лімфогранулематоз Lymphogranulomatosis

Лімфома Беркітта Burkitt's lymphoma

Неходжкінська лімфома Non-Hodgkin lymphoma

Хронічний лімфолейкоз Chronic lymphocytic leukemia

Гострий лейкоз Acute leukemia

91 / 130
У дівчинки 3-х років через 1 годину після вживання у їжу шоколадно-горіхового крему на шкірі з’явилась рясна уртикарна висипка із вираженим свербежем. Діагностовано гостру кропив’янку. Які імунопатологічні реакції лежать в основі даного захворювання? In a 3-year-old girl, 1 hour after eating chocolate-nut cream, a profuse urticarial rash with severe itching appeared on her skin. Diagnosed with acute urticaria' Yanku. What immunopathological reactions underlie this disease?

Аутоімунні Autoimmune

Цитотоксичні Cytotoxic

Негайного типу Immediate type

Уповільненого типу Delayed type

Імунокомплексні Immune complex

92 / 130
До лікаря звернувся чоловік 36-ти років зі скаргами на пекучий загрудинний біль, що зазвичай виникає через 1-1,5 години після вживання їжі. Зазначає, що біль підсилюється в горизонтальному положенні. При ендоскопії верхнього відділу ШКТ виявлені 2 вогнищевих ураження нижньої третини слизової оболонки стравоходу до 5 мм в межах однієї складки. Яка тактика лікаря буде найбільш доречною? A 36-year-old man went to the doctor with complaints of burning chest pain, which usually occurs 1-1.5 hours after eating. He notes that the pain intensifies in a horizontal position. Endoscopy of the upper part of the gastrointestinal tract revealed 2 focal lesions of the lower third of the esophageal mucosa up to 5 mm within one fold. What tactics of the doctor would be most appropriate?

Хірургічне лікування Surgical treatment

Призначення ненаркотичних анальгетиків Prescription of non-narcotic analgesics

Призначення інгібіторів протонної помпи Prescription of proton pump inhibitors

Призначення кларитроміцину Clarithromycin Prescription

93 / 130
Після потрапляння розчину гашеного вапна в ліве око у пацієнта vis OS=0,05. Повіки та кон’юнктива червоні, набряклі. Змішана ін’єкція, хемоз. Рогівка сірого кольору, набрякла, тьмяна, напівпрозора. Який характер опіку лівого ока? After getting a slaked lime solution into the patient's left eye, vis OS=0.05. Eyelids and conjunctiva are red, swollen. Mixed injection, chemosis. The cornea is gray, swollen, dull, translucent. What is the nature of the burn in the left eye?

Хімічний опік рогівки та кон’юнктиви першого ступеня лівого ока (гашеним вапном) Chemical burn of the cornea and conjunctiva of the first degree of the left eye (with slaked lime)

Термічний опік рогівки та кон’юнктиви першого ступеня лівого ока (гашеним вапном) Thermal burn of the cornea and conjunctiva of the first degree of the left eye (with slaked lime)

Хімічний опік рогівки та кон’юнктиви третього ступеня лівого ока (гашеним вапном) Chemical burn of the cornea and conjunctiva of the third degree of the left eye (with slaked lime)

Хімічний опік рогівки та кон’юнктиви другого ступеня лівого ока (гашеним вапном) Chemical burn of the cornea and conjunctiva of the second degree of the left eye (with slaked lime)

Післятравматичний кератит лівого ока Post-traumatic keratitis of the left eye

94 / 130
На прийом до лікаря акушера-гінеколога жіночої консультації звернулася жінка зі строком вагітності 10 тижнів (перше звертання). Який з медичних документів повинен бути заведений лікарем у цьому випадку для контролю за перебігом вагітності? A woman with a 10-week pregnancy period applied to see an obstetrician-gynecologist at the women's consultation (first visit). Which of the medical documents should be prepared by the doctor in this case for pregnancy control?

Карта обліку диспансеризації Dispensary accounting card

Контрольна карта диспансерного спостереження Dispensary monitoring control card

Індивідуальна карта вагітної, породіллі Individual card of a pregnant woman, a woman giving birth

Історія пологів History of childbirth

Медична карта амбулаторного хворого Medical card of an outpatient

95 / 130
У роділлі з регулярною пологовою діяльністю відійшли навколоплідні води зеленого кольору. Серцебиття плода 150/хв. Голівка плода притиснута до входу в малий таз. Піхвове дослідження: шийка матки відкрита до 5 см, передлежить голівка, мале тім’ячко розташоване ліворуч під симфізом. Що має бути використане для моніторингу стану плода? In a woman in labor with regular labor, green amniotic fluid has passed. Fetal heart rate 150/min. The fetal head is pressed against the entrance to the small pelvis. Vaginal examination: the cervix is open up to 5 cm, the head is presented, the small crown is located on the left under the symphysis. What should be used to monitor the condition of the fetus?

Амніоскопія Amnioscopy

Кардіотокографія Cardiotocography

Визначення pH навколоплідних вод Determination of amniotic fluid pH

Біофізичний профіль плода Biophysical profile of the fetus

Доплерометрія пуповинного кровотоку Dopplerometry of umbilical cord blood flow

96 / 130
Мати хлопчика 6-ти місяців прийшла на прийом до педіатра стурбована тим, що її дитина не отримувала жодних щеплень. Скарг не мають. Об’єктивно: температура - 37,1°С, частота дихання - 20/хв., артеріальний тиск - 100/70 мм рт.ст. Зріст та вага відповідають діапазону між 50 та 75 перцентилем. Відповідно до нормативно-правових документів МОЗ, якої тактики має дотримуватися лікар щодо імунізації цієї дитини? The mother of a 6-month-old boy came to see a pediatrician worried that her child had not received any vaccinations. They have no complaints. Objectively: temperature - 37 ,1°C, respiratory rate - 20/min, blood pressure - 100/70 mm Hg. Height and weight correspond to the range between the 50th and 75th percentile. According to the regulatory and legal documents of the Ministry of Health, what tactics should a doctor follow regarding immunization of this child?

Ввести вакцину БЦЖ, КПК та АКДС Inject BCG, CPC and DPT vaccine

Ввести вакцину БЦЖ Enter BCG vaccine

Провести пробу Манту та записати на прийом через 2-3 дні Conduct a Mantoux test and make an appointment in 2-3 days

Відкласти вакцинацію та записати на прийом через тиждень Postpone the vaccination and make an appointment in a week

97 / 130
Хворий 47-ми років госпіталізований в клініку в тяжкому стані із скаргами на біль у черевній порожнині, слабкість, запаморочення. Одну годину назад отримав сильний удар в ліву половину живота, свідомість не втрачав. При огляді - блідість шкіри та слизових оболонок. Пульс - 120/хв., AT- 80/40 мм рт.ст. Живіт при пальпації напружений, болючий зліва від пупка. Симптом Щоткіна-Блюмберга позитивний. Сеча без змін. Загальний аналіз крові: еритроцити -2,8- 1012/л, НЬ- 90 г/л. Яке пошкодження можна припустити в першу чергу? A 47-year-old patient was hospitalized in a serious condition with complaints of pain in the abdominal cavity, weakness, dizziness. One hour ago, he received a strong blow to the left half of the abdomen , did not lose consciousness. On examination - pallor of the skin and lacrimal membranes. Pulse - 120/min., AT - 80/40 mm Hg. Abdomen on palpation is tense, painful to the left of the navel. Schottkin-Blumberg symptom is positive. Urine without changes. General blood analysis: erythrocytes -2.8- 1012/l, Hb- 90 g/l. What kind of damage can be assumed in the first place?

Пошкодження нирки Kidney damage

Субкапсулярний розрив селезінки Subcapsular rupture of the spleen

Розрив товстої кишки. Каловий перитоніт Rupture of the colon. Fecal peritonitis

Закрита травма черевної порожнини без пошкодження внутрішніх органів Closed trauma of the abdominal cavity without damage to internal organs

Розрив селезінки. Внутрішня кровотеча Rupture of the spleen. Internal bleeding

98 / 130
У 2-місячної дитини протягом останнього місяця визначаються пароксизми дратівливості, неспокою та крику, які починаються без очевидної причини. Напади виникають під час або відразу після годування. Помітне полегшення часто наступає після дефекації. Тривалість епізодів близько трьох годин на день. Між нападами дитина спокійна, має добрий апетит, прибавка в масі тіла фізіологічна. Який найбільш імовірний діагноз? A 2-month-old child has had paroxysms of irritability, restlessness, and screaming during the last month that begin for no apparent reason. Attacks occur during or immediately after feeding. Marked relief often occurs after defecation. The duration of episodes is about three hours a day. Between attacks, the child is calm, has a good appetite, the increase in body weight is physiological. What is the most likely diagnosis?

Малюкові кольки Baby colic

Функціональна диспепсія Functional dyspepsia

Дитяча дишезія Child Dyshesia

Функціональний закреп Functional Crap

Лактазна недостатність Lactase deficiency

99 / 130
Хвора 26-ти років перебуває в шлюбі 5 років. За відсутності контрацепції та наявності регулярного статевого життя вагітність відсутня, чоловік обстежений, фертильний. З анамнезу: в 19 років оперована з приводу явищ розлитого перитоніту. Які дослідження необхідно провести для з’ясування причин безпліддя? The 26-year-old patient has been married for 5 years. In the absence of contraception and regular sex life, there is no pregnancy, the husband has been examined, he is fertile. From the anamnesis: at the age of 19 was operated on due to the phenomena of spilled peritonitis. What studies should be conducted to find out the causes of infertility?

Гістеросальпінгографія або лапароскопія Hysterosalpingography or laparoscopy

Гістероскопія з оцінкою функціонального стану ендометрію Hysteroscopy with evaluation of the functional state of the endometrium

Дослідження статевих стероїдів, гонадотропінів, фолікулометрія Research of sex steroids, gonadotropins, folliculometry

Кімографічна пертубація маткових труб Kymographic pertubation of fallopian tubes

Тести функціональної діагностики Functional diagnostic tests

100 / 130
У щойно народженої доношеної дитини, якій надають реанімаційну допомогу, після початкового періоду штучної вентиляції легень мішком і маскою ЧСС становить 50/хв. Звертають також на себе увагу тотальний ціаноз і відсутність самостійного дихання. Яким має бути наступний реанімаційний крок? After the initial period of artificial ventilation with a bag and a mask, the heart rate is 50/min in a newborn full-term child who is being resuscitated. Total cyanosis is also noted and no spontaneous breathing. What should be the next resuscitation step?

Катетеризувати вену пуповини і ввести адреналін Catheterize the umbilical vein and inject adrenaline

Продовжити штучну вентиляцію легень Continue artificial ventilation

Інтубувати трахею, ввести ендотра-хеально адреналін і відновити штучну вентиляцію легень Intubate the trachea, inject epinephrine endotracheally and resume artificial ventilation

Розпочати непрямий масаж серця і відновити штучну вентиляцію легень Start indirect heart massage and resume artificial ventilation

Призначити кисневу терапію Prescribe oxygen therapy

101 / 130
Пацієнт 37-ми років надійшов у нефрологічне відділення з діагнозом: хронічна хвороба нирок, І ст. - мезангіопроліферативний гломерулонефрит. АТ- 140/90 мм рт.ст. Добова втрата білка із сечею становить - 5,1 У крові: загальний білок - 48, г/л; сечовина - 5,7 ммоль/л; креатинін - 76,3 мкмоль/л; холестерин - 8,1 ммоль/л. Яку групу препаратів необхідно призначити? A 37-year-old patient was admitted to the nephrology department with a diagnosis of chronic kidney disease, stage I - mesangioproliferative glomerulonephritis. Blood pressure - 140/90 mmHg. The daily loss of protein with urine is - 5.1 In the blood: total protein - 48 g/l, urea - 5.7 mmol/l, creatinine - 76.3 μmol/l, cholesterol - 8.1 mmol/l. a group of drugs must be prescribed?

Нестероїдні протизапальні Nonsteroidal anti-inflammatory drugs

Сульфаніламідні Sulfonamides

Антибіотики Antibiotics

Нітрофурани Nitrofurans

Глюкокортикостероїди Glucocorticosteroids

102 / 130
Пацієнтка, що 20 років страждає на параноїдну форму шизофренії, звернулася до дільничного психіатра зі скаргами на появу ’’голосів” у голові, що погрожують, лаються, безсоння, тривогу, думки, що ’’сусіди за нею стежать’.’ Препарати якої групи доцільно призначити? A patient suffering from a paranoid form of schizophrenia for 20 years turned to a district psychiatrist with complaints about the appearance of 'voices' in her head that threaten, swear, insomnia, anxiety, thoughts that 'neighbors are watching her.' What group of drugs should be prescribed?

Ноотропні засоби Nootropics

Анксіолітики Anxiolytics

Антидепресанти Antidepressants

Нормотиміки Normothymics

Антипсихотичні засоби Antipsychotics

103 / 130
Пацієнт перебував на стаціонарному лікуванні в терапевтичному відділенні з приводу загострення хронічного бронхіту з 3.04 по 12.04. Був виписаний зі стаціонару. Листок непрацездатності було видано за підписом лікуючого лікаря. Чи правильно було проведено експертизу тимчасової втрати працездатності? The patient was hospitalized in the therapeutic department due to an exacerbation of chronic bronchitis from April 3 to April 12. He was discharged from the hospital. The sick leave was issued under the signature of the attending physician. was the examination of temporary disability properly conducted?

Ні, крім лікуючого лікаря має бути підпис заступника головного лікаря міської лікарні No, in addition to the attending physician, there must be a signature of the deputy chief physician of the city hospital

Ні, крім лікуючого лікаря має бути підпис завідувача терапевтичного відділення No, in addition to the attending physician, there must be a signature of the head of the therapeutic department

Так, підпису лікуючого лікаря було достатньо Yes, the signature of the attending physician was sufficient

Ні, листок непрацездатності має бути підписаний тільки завідувачем терапевтичного відділення No, the sick leave sheet must be signed only by the head of the therapeutic department

Ні, крім лікуючого лікаря має бути підпис головного лікаря міської лікарні No, in addition to the attending physician, there must be a signature of the chief physician of the city hospital

104 / 130
В місті N проводилося вивчення захворюваності на інфаркт міокарда за попередні роки (з 2009 по 2013 роки). Який вид епідеміологічного дослідження був використаний? In the city N, the incidence of myocardial infarction was studied in previous years (from 2009 to 2013). What type of epidemiological study was used?

Описовий Descriptive

Аналітичний Analytical

Ретроспективний Retrospective

Проспективний Prospective

Експериментальний Experimental

105 / 130
Хворий 35-ти років впродовж 14-ти років працює в ливарному цеху, де концентрація кварцевого пилу в 4 рази перевищує ГДВ продовж 4-х років його турбує кашель, задишка при фізичному навантаженні. При аускультації дихання ослаблено. Рентгенографія ОГК: емфізема, дрібно-плямисті тіні по всім легеневим полям. Найбільш імовірний діагноз? A 35-year-old patient has been working in a foundry for 14 years, where the concentration of quartz dust is 4 times higher than the GDV. For 4 years, he has been bothered by cough, shortness of breath during physical exertion. Upon auscultation, breathing is weakened. X-ray of the chest cavity: emphysema, small-spotted shadows in all lung fields. The most likely diagnosis?

Туберкульоз легенів Pulmonary tuberculosis

Хронічний бропхіт Chronic bronchitis

Азбестоз Asbestosis

Силікоз Silicosis

ХОЗЛ COPD

106 / 130
Хвора 54-х років скаржиться на мерзлякуватість, сонливість, слабкість. В минулому лікувалася з приводу аутоімунного вузлового зоба. За рік її стан погіршився: з’явилася слабкість, набрала вагу, стала повільною. Щитоподібна залоза - I ст. Шкіра холодна, суха, бліда, язик потовщений. Іолос захриплий. Тони серця ритмічні, глухі, частота серцевих скорочень - 56/хв., живіт здутий. Випорожнення - закрепи. Який препарат слід призначити для профілактики значного збільшення щитоподібної залози? A 54-year-old patient complains of chills, drowsiness, weakness. In the past, she was treated for autoimmune nodular goiter. Over the course of a year, her condition worsened: weakness appeared, gained weight, became sluggish. Thyroid gland - 1st stage. Skin cold, dry, pale, tongue thickened. Iolos hoarse. Heart sounds are rhythmic, dull, heart rate - 56/min., abdomen is distended. Stools - constipation. What drug should be taken appoint for the prevention of a significant increase in the thyroid gland?

Преднізолон Prednisone

Радіоактивний йод Radioactive iodine

Мерказоліл Mercazolil

Тироксин Tyroxin

Метилтіоурацил Methylthiouracil

107 / 130
У дитини 9-ти років виник біль у верхній третині правої гомілки, підвищилась температура до 39oC, не може стати на ногу. В анамнезі мав травму гомілки та переніс ангіну. Який найбільш імовірний діагноз? A 9-year-old child developed pain in the upper third of the right shin, the temperature rose to 39oC, he could not stand up. He had a history of shin injury and angina . What is the most likely diagnosis?

Туберкульозний остеомієліт Tuberculous osteomyelitis

Перелом кістки Bone fracture

Злоякісна пухлина Malignant tumor

Гострий гематогенний остеомієліт Acute hematogenous osteomyelitis

Гострий ревматизм Acute rheumatism

108 / 130
Хвора 64-х років скаржиться на загальну слабкість, шум у голові, осиплість голосу. Об’єктивно: блідість з жовтяничним відтінком, язик червоний зі згладженими сосочками, асиметрія тактильної і больової чутливості. Пульс - 120/хв., артеріальний тиск - 80/50 мм рт.ст. Пальпується селезінка. У крові: НЬ- 58 г/л, еритроцити -1,24*1012/л, лейкоцити - 2,8-109/л, тромбоцити - 140 * 109/л, швидкість осідання еритроцитів - 17 мм/год, анізоцитоз, пойкілоцитоз - виражені (++). Яке дослідження буде вирішальним у з’ясуванні генезу анемії? A 64-year-old patient complains of general weakness, noise in the head, hoarseness of voice. Objectively: paleness with a jaundiced tint, red tongue with flattened papillae, asymmetry tactile and pain sensitivity. Pulse - 120/min., blood pressure - 80/50 mm Hg. The spleen is palpable. In the blood: Hb - 58 g/l, erythrocytes - 1.24*1012/l, leukocytes - 2, 8-109/l, platelets - 140 * 109/l, erythrocyte sedimentation rate - 17 mm/h, anisocytosis, poikilocytosis - pronounced (++). Which study will be decisive in clarifying the genesis of anemia?

Фіброгастроскопія Fibrogastroscopy

Стернальна пункція Sternal puncture

Пряма проба Кумбса Direct Coombs test

Люмбальна пункція Lumbar puncture

Непряма проба Кумбса Indirect Coombs Test

109 / 130
Хворому 65-ти років на гіпертонічну хворобу з супутньою ГХС, призначена фармакотерапія. Через 2 тижні звернувся до лікаря з приводу набряків на гомілках. Під час обстеження не було виявлено ознак захворювань, які перебігають з набряковим синдромом. Припущено побічну дію лікарського засобу. Який з наве дених препаратів був вилучений зі схеми лікування? A 65-year-old patient is prescribed pharmacotherapy for hypertensive disease with concomitant HCV. After 2 weeks, he consulted a doctor about swelling on the lower legs. During the examination, no symptoms were found signs of diseases that occur with edema syndrome. A side effect of the drug is assumed. Which of the following drugs was removed from the treatment regimen?

Бісопролол Bisoprolol

Ацетилсаліцилова кислота Acetylsalicylic acid

Аторвастатин Atorvastatin

Амлодипін Amlodipine

Раміприл Ramipril

110 / 130
Вам довелося прийти на допомогу жінці 58-ми років, у якої виникла рясна кровотеча з розірваного варикозно розширеного венозного вузла на лівій гомілці. У чому полягає перша допомога? You had to attend to a 58-year-old woman who was bleeding profusely from a ruptured varicose vein in her left leg. What is first aid?

Z-подібний шов на розірваний варикозний вузол Z-shaped suture on a ruptured varicose node

Накладання джгута проксимально до джерела кровотечі Applying a tourniquet proximal to the source of bleeding

Накладання джгута дистально до джерела кровотечі Applying a tourniquet distal to the source of bleeding

Операція Троянова-Тренделенбурга Troyanov-Trendelenburg operation

Підвищене положення кінцівки. Стискальна стерильна пов’язка Elevated limb position. Compression sterile bandage

111 / 130
Хвора 35-ти років скаржиться на перебої в роботі серця, зниження ваги, біль передньої поверхні шиї, що почалися після ГРВІ. Пульс - 110/хв., АТ- 130/60 мм рт.ст. Щитоподібна залоза незначно збільшена, болюча при пальпації. ШОЕ-40 мм/год. ТТГ- 0,05 мОд/л, вТ4 - 34,3 пмоль/л. Який пайбільш імовірний діагноз? A 35-year-old patient complains of heart failure, weight loss, pain in the front surface of the neck that started after SARS. Pulse - 110/min., BP - 130/60 mm Hg. The thyroid gland is slightly enlarged, painful on palpation. ESR-40 mm/h. TSH-0.05 mU/l, vT4 - 34.3 pmol/l. What is the most likely diagnosis?

Гострий тиреоїдит Acute thyroiditis

Дифузний токсичний зоб Diffuse toxic goiter

Пароксизмальна тахікардія Paroxysmal tachycardia

Тиреоїдит Хашимото Hashimoto's thyroiditis

Підгострий тиреоїдит Subacute thyroiditis

112 / 130
Хворий 75-ти років перебуває на консервативному лікуванні у відділенні судинної неврології з приводу ТІА. Згідно з останніми даними кольорового дуплексного сканування у пацієнта наявний стеноз лівої внутрішньої сонної артерії гомогенною атеросклеротичною бляшкою більше 70%. Судинний хірург рекомендував пацієнту проведення опера тивного втручання. Про яке оперативне втручання йде мова? A 75-year-old patient is undergoing conservative treatment in the Department of Vascular Neurology for a TIA. According to the latest data of a color duplex scan, the patient has a homogeneous stenosis of the left internal carotid artery with an atherosclerotic plaque of more than 70%. The vascular surgeon recommended the patient to undergo surgery. What kind of surgery are we talking about?

Ендартеректомія з лівої внутрішньої сонної артерії Endarterectomy from the left internal carotid artery

Перев’язка лівої внутрішньої сонної артерії Left internal carotid artery ligation

Тромбектомія з лівої внутрішньої сонної артерії Thrombectomy from the left internal carotid artery

Магістралізація лівої зовнішньої сонної артерії Left external carotid artery trunking

Підключично-зовнішньосонне шунтування зліва Left subclavian-external carotid shunt

113 / 130
Хворий 25-ти років надійшов через 2 години після дорожньо-транспортної аварії зі скаргами на постійний інтенсивний біль у правій паховій ділянці та у симфізі тазу, неможливість самостійно пересуватися. При огляді: положення хворого вимушене - поза ”жаби” (позитивний симптом Волковича). Садна й синці на шкірі в проекції симфізу та горизонтальної гілки лобкової кістки праворуч, виражений набряк м’яких тканин. Позитивний симптом 'прилиплої п’яти” праворуч. Поставте попередній діагноз: A 25-year-old patient arrived 2 hours after a traffic accident with complaints of constant intense pain in the right inguinal area and pelvic symphysis, inability to move independently. During the examination: the patient's position is forced - the 'frog' position (positive Volkovich's symptom). Bruises and bruises on the skin in the projection of the symphysis and the horizontal branch of the pubic bone on the right, pronounced swelling of soft tissues. Positive symptom of 'sticky heel' on the right. Put previous diagnosis:

Травматична пахвинна грижа Traumatic inguinal hernia

Забій кісток тазу Pelvic fracture

Закритий перелом лобкової кістки праворуч Closed fracture of pubic bone on the right

Травматична аневризма стегнової артерії Traumatic femoral artery aneurysm

Вивих правого стегна Dislocation of right hip

114 / 130
Підліток госпіталізований зі скаргами на обмеженість рухів у нижніх кінцівках. Захворювання розпочалось 2 дні тому з підвищення температури тіла до 38,3°С, рідких випорожнень 3-4 рази на добу. Об’єктивно: температура тіла -36,8°С, активні рухи в нижніх кінцівках відсутні, у зоні ураження - арефлексія, гіпотонія м’язів, чутливість збережена. Менінгеальні симптоми слабко позитивні. Яке захворювання можна припустити? A teenager was hospitalized with complaints of limited movement in the lower limbs. The disease began 2 days ago with an increase in body temperature to 38.3°C, loose stools 3-4 times per day. Objectively: body temperature is -36.8°C, there are no active movements in the lower limbs, areflexia, muscle hypotonia, sensitivity is preserved in the affected area. Meningeal symptoms are weakly positive. What disease can be assumed?

Поліомієліт Polio

Герпетичний менінгоенцефаліт Herpetic meningoencephalitis

Травма поперекового відділу хребта Injury of the lumbar spine

Субарахноїдальний крововилив Subarachnoid hemorrhage

Ботулізм Botulism

115 / 130
У хворого 25-ти років протягом 6 тижнів з’явився біль в крижах, попереку, ахілових сухожилках, ранкова скутість до півгодини. В аналізі крові: ШОЕ-30 мм/год; СРВ- 25 мг/л; РФ - норма, HLA В27 - позитивний. МРТ попереково-крижового відділу: двобічний сакроілеїт 2 ст. Який найбільш імовірний діагноз? A 25-year-old patient developed pain in the sacrum, lower back, and Achilles tendons for 6 weeks, morning stiffness for up to half an hour. Blood test: ESR-30 mm/h; SRV - 25 mg/l; RF - normal, HLA B27 - positive. MRI of the lumbosacral region: bilateral sacroiliitis of the 2nd degree. What is the most likely diagnosis?

Анкілозуючий спондиліт Ankylosing spondylitis

Реактивний артрит Reactive arthritis

Подагричний артрит Gouty arthritis

Ревматоїдний артрит Rheumatoid arthritis

Остеохондроз поперекового відділу Osteochondrosis of the lumbar region

116 / 130
У жінки 30-ти років було три самовільних викидня та одні передчасні пологи. При обстеженні має місце позитивна реакція на антікардіоліпінові антитіла та вовчаковий антикоагулянт. Поставте діагноз: A 30-year-old woman had three spontaneous miscarriages and one premature birth. During the examination, there is a positive reaction to anticardiolipin antibodies and lupus anticoagulant. Make a diagnosis:

Геморагічний синдром Hemorrhagic syndrome

Синдром Штейна-Левенталя Stein-Leventhal syndrome

Синдром Шихана Sheehan Syndrome

Антифосфоліпідний синдром Antiphospholipid syndrome

Синдром тестикулярної фемінізації Testicular feminization syndrome

117 / 130
В органiзм людини з атмосферного повiтря надходять декiлька хiмiчних речовин. Як називається тип комбiнованої дiї, при якому сумiсний ефект менший вiд суми ефектiв кожної з речовин, що входить у комбiнацiю, при їх iзольованiй дiї на органiзм? Several chemical substances enter the human body from the atmospheric air. What is the type of combined action called, in which the combined effect is less than the sum of the effects of each of the substances included in the combination , with their isolated effect on the organism?

Iзольована дiя Isolated action

Потенцiювання Potentiation

Антагонiзм Antagonism

Комплексна дiя Complex action

Поєднана дiя Combined action

118 / 130
Хвора 65-ти років госпіталізована в реанімаційне відділення в коматозному стані. Хворіє на цукровий діабет ІІ-го типу 10 років. Останні 2 тижні спостерігалися виражена поліурія, полідипсія. Глюкоза сироватки крові - 30 ммоль/л, рН артеріальної крові - 7,3. Осмолярність плазми крові - 350 мОсм/л. Хворій встановлено діагноз: діабетична гіперосмолярна кома. Який основний патогенетичний механізм коми? A 65-year-old patient was hospitalized in the intensive care unit in a comatose state. She has been suffering from type II diabetes for 10 years. In the last 2 weeks, severe polyuria and polydipsia were observed. Blood serum glucose - 30 mmol/l, arterial blood pH - 7.3. Blood plasma osmolality - 350 mOsm/l. The patient was diagnosed with diabetic hyperosmolar coma. What is the main pathogenetic mechanism of coma?

Гіпонатріємія Hyponatremia

Дегідратація Dehydration

Підвищення канальцевої реабсорбції Increasing tubular reabsorption

Гіпокоагуляція Hypocoagulation

Підвищення клубочкової фільтрації Increasing glomerular filtration

119 / 130
Чоловiк 32-х рокiв звернувся до лiкаря зi скаргами на висип, що з’явився на колiнах 4 мiсяцi тому i зараз вперше починає з’являтися на лiктях. Зi слiв пацiєнта болю не вiдчуває, але мiсце висипу злегка свербить та кровоточить при розчiсуваннi. Вiн вiдзначив, що на лiктях висип з’явився пiсля подряпин, якi йому нанiс його домашнiй кiт. При фiзикальному обстеженнi лiкарем виявлено на колiнах та лiктях еритематознi бляшки дiаметром вiд 2 до 3 см iз чiтким контуром, щiльнi, вкритi срiблястими лусочками. Який дiагноз є найбiльш iмовiрним? A 32-year-old man went to the doctor complaining of a rash that appeared on his knees 4 months ago and is now starting to appear on his fingers for the first time. the patient says he does not feel pain, but the rash is slightly itchy and bleeds when combed. He noted that the rash appeared on his fingers after being scratched by his pet cat. During a physical examination, the doctor revealed erythematous plaques on the knees and fingers with a diameter of 2 up to 3 cm with a clear contour, dense, covered with silvery scales. What diagnosis is most likely?

Себорейний дерматит Seborrheic dermatitis

Екзема Eczema

Контактний дерматит Contact dermatitis

Хвороба котячих подряпин Cat Scratch Disease

Псорiаз Psoriasis

120 / 130
Хворий 20-ти років пройшов курс поліхі-міотерапії за схемою ВАМП з приводу гострого лімфобластного лейкозу. Яка морфологічна картина кісткового мозку може свідчити про настання ремісії? A 20-year-old patient underwent a course of polychemotherapy according to the VAMP scheme for acute lymphoblastic leukemia. What morphological picture of the bone marrow can indicate the onset of remission?

Відсутність бластних клітин No blast cells

Вміст бластних клітин до 10% Content of blast cells up to 10%

Вміст бластних клітин до 1% Content of blast cells up to 1%

Вміст бластних клітин до 5% Content of blast cells up to 5%

Вміст бластних клітин до 15 % Content of blast cells up to 15%

121 / 130
Жінка 56-ти років звернулася до лікаря зі скаргами на підвищення температури до 38,5°С, підвищену пітливість вночі та збільшення лімфовузлів в лівій пахвовій та лівій надключичній ділянках. У сімейному анамнезі рак молочної залози у бабусі. При фізикальному обстеженні в зазначених ділянках пальпуються збільшені, безболісні, щільні лімфовузли. При рентгенологічному дослідженні органів грудної порожнини розширення тіні середостіння за рахунок збільшених лімфовузлів. Після ексцизійної біопсії лімфовузла надключичної ділянки, виявлені атипові багатоядерні клітини Рід-Штернберга. Який діагноз є найбільш імовірним? A 56-year-old woman consulted a doctor with complaints of a temperature rise to 38.5°C, increased sweating at night, and an increase in lymph nodes in the left axillary and left supraclavicular areas . There is a family history of breast cancer in the grandmother. During the physical examination, enlarged, painless, dense lymph nodes are palpated in the indicated areas. During the X-ray examination of the chest cavity, the expansion of the mediastinal shadow due to enlarged lymph nodes. After an excisional biopsy of the supraclavicular lymph node, atypical multinucleated Reed cells were detected - Sternberg. Which diagnosis is the most probable?

Метастази раку молочної залози Breast cancer metastases

Бронхогенна карцинома Bronchogenic carcinoma

Лімфома Ходжкіна Hodgkin lymphoma

Туберкульоз Tuberculosis

Інфекційний мононуклеоз Infectious mononucleosis

122 / 130
Хворий 54-х років звернувся до сімейного лікаря зі скаргами на утруднення дихання, слабкість. Останпі два тижні турбував біль та набряк правої нижньої кінцівки. Дані симптоми вперше в житті, раніше на обліку у лікаря не перебував. АТ- 110/70 мм ртст, ЧСС-96/хЯкий діагностичний метод має вирішальне значення? A 54-year-old patient turned to his family doctor with complaints of difficulty breathing, weakness. For the past two weeks, he has been troubled by pain and swelling of the right lower limb. These symptoms are the first time in his life , had not been registered with a doctor before. Blood pressure - 110/70 mm Hg, heart rate - 96/hWhich diagnostic method is crucial?

Функція зовнішнього дихання External breathing function

Рентгенографія органів грудної клітки X-ray of chest organs

Ехокардіографія Echocardiography

Електрокардіографія Electrocardiography

Мультиспіральна КТ ангіографія Multispiral CT angiography

123 / 130
Хвора 72-х років звернулася до лікаря зі скаргами на наявність новоутворення на шкірі правої щоки, яке виявила самостійно близько 6 місяців тому. Об’єктивно: на шкірі правої щоки виразка неправильної форми з підритими краями та нечіткими контурами діаметром 1,5x2,0 см, багряно-червоного кольору, контактно кровить. Встановлено діагноз: підозра на рак шкіри правої щоки. Який метод верифікації діагнозу найбільш доцільний у даному випадку? A 72-year-old patient turned to the doctor with complaints about the presence of a neoplasm on the skin of the right cheek, which she discovered on her own about 6 months ago. Objectively: on the skin of the right cheeks, an ulcer of an irregular shape with undercut edges and indistinct contours with a diameter of 1.5x2.0 cm, purple-red color, bleeding on contact. The diagnosis is established: suspected cancer of the skin of the right cheek. What method of verifying the diagnosis is most appropriate in this case?

Пункційна тонкоголкова аспіраційна біопсія Puncture fine-needle aspiration biopsy

Інцизійна біопсія Incisional biopsy

Трепан-біопсія Trepan biopsy

Мазок-відбиток Smear-imprint

Зішкріб пухлини Scraping tumor

124 / 130
У хворого 35-ти років, що страждає на хронічний гломерулонефрит і останні 3 роки перебуває на гемодіалізі, з’явилися перебої в роботі серця, гіпотонія, зростаюча слабкість, задишка. На ЕКГ: брадикардія, атріовентрикулярна блокада I ст., високі загострені зубці Т. Напередодні - грубе порушення питного та дієтичного режимів. Яка найбільш імовірна причина вказаних змін? A 35-year-old patient who suffers from chronic glomerulonephritis and has been on hemodialysis for the past 3 years has had heart failure, hypotension, increasing weakness, shortness of breath. On the ECG: bradycardia, atrioventricular block of the 1st degree, high pointed T waves. The day before - a gross violation of drinking and dietary regimes. What is the most likely cause of the indicated changes?

Гіпергідратація Hyperhydration

Гіперкаліємія Hyperkalemia

Гіпернатріємія Hypernatremia

Гіпокальціємія Hypocalcemia

Гіпокаліємія Hypokalemia

125 / 130
Хлопчика 2-х років госпіталізовано з приводу зменшення маси тіла, нестійких випорожнень, анорексії, які з’явилися після введення в раціон манної каші (з 5 місяців). Дитина адинамічна, млява, шкіра бліда, суха, підшкірно-жировий шар відсутній. Живіт здутий, напружений. Під час перкусії у верхній частині живота тимпаніт, шум плеску, випорожнення пінисті, світлого кольору, смердючі. В копроцитограмі: нейтральний жир - багато. Який наступний крок у веденні пацієнта буде найбільш доречним? A 2-year-old boy was hospitalized due to a decrease in body weight, unstable stools, anorexia, which appeared after the introduction of semolina into the diet (from 5 months). The child is adynamic, lethargic, the skin is pale, dry, the subcutaneous fat layer is absent. The abdomen is distended, tense. During percussion in the upper part of the abdomen, tympanitis, splashing noise, stools are foamy, light-colored, smelly. In the coprocytogram: neutral fat - a lot. What What would be the most appropriate next step in patient management?

Визначення IgA до тканинної транс-глютамінази Determination of IgA to tissue trans-glutaminase

Призначення антибіотиків широкого спектру дії Prescription of broad-spectrum antibiotics

Негайне оперативне втручання Immediate operative intervention

Рентгенографія органів черевної порожнини X-ray of abdominal organs

Розробка індивідуального плану харчування з великим вмістом клітковини Development of an individual meal plan with high fiber content

126 / 130
У дитини 13-ти років сполучена вада мі-трального клапана з переважанням недостатності. На тлі гострої респіраторної вірусної інфекції (ГРВІ) підсилилися задишка та загальна слабкість, з’явилося відчуття стискання в грудях, сухий кашель. В легенях вислуховуються різнокаліберні вологі хрипи. Печінка біля краю реберної дуги. Який невідкладний стан розвинувся у дитини? A 13-year-old child has a combined defect of the mitral valve with a predominance of insufficiency. Against the background of an acute respiratory viral infection (ARSI), shortness of breath and general weakness increased, with' there was a feeling of tightness in the chest, a dry cough. Wet rales of various caliber are heard in the lungs. The liver is near the edge of the costal arch. What emergency condition has developed in the child?

Гостра правошлуночкова серцева недостатність Acute right ventricular heart failure

Гостра тотальна серцева недостатність Acute total heart failure

Гостра судинна недостатність Acute vascular insufficiency

Гостра лівошлуночкова серцева недостатність Acute left ventricular heart failure

Хронічна лівошлуночкова серцева недостатність Chronic left ventricular heart failure

127 / 130
Студентка медичного коледжу (вік 17,8 років) влаштовується на роботу молодшою медичною сестрою в рентгенкабінет поліклініки. Чи може вона працювати на цій посаді? A medical college student (age 17.8 years old) gets a job as a junior nurse in the x-ray room of a polyclinic. Can she work in this position?

Можна, після проведення медичного обстеження It is possible, after a medical examination

Можна, із застосуванням індивідуальних засобів захисту It is possible, with the use of individual protection

Hi Hi

Можна без обмежень No restrictions

Можна, зі скороченою тривалістю робочого дня It is possible, with a shortened working day

128 / 130
Вагітній 20-ти років в терміні 5 тижнів лікар жіночої консультації призначив повноцінне харчування та вживання фолієвої кислоти 0,4 мг. Профілактика якого ускладнення проводиться? For 5 weeks, a 20-year-old pregnant woman was prescribed a full-fledged diet and the use of folic acid 0.4 mg. Which complication is being prevented?

Переношування вагітності Pregnancy carryover

Прееклампсії Preeclampsia

Затримки розвитку плода Fetal development delays

Вад розвитку нервової системи Defects of the nervous system

Дистресу плода Fetal distress

129 / 130
До сімейного лікаря звернувся пацієнт з астено-вегетативним синдромом. Пацієнт пред’являв скарги на порушення сну, частин головний біль, запаморочення, дратівливість, підвищену втомлюваність, порушення пам’яті. Пацієнт протягом 2-х років не працював, проживає в житловому будинку, який розташований поруч з аеропортом. З впливом якого чинника найімовірніше можливе виникнення даних симптомів? A patient with astheno-vegetative syndrome consulted a family doctor. The patient complained of sleep disturbances, headache, dizziness, irritability, increased fatigue, memory impairment 'yati. The patient has not worked for 2 years, lives in a residential building located near the airport. What factor is most likely to cause these symptoms?

Електромагнітні поля Electromagnetic fields

Хімічне забруднення атмосферного повітря Chemical air pollution

Ультразвук Ultrasound

Шум Noise

Вібрація Vibration

130 / 130
Хворий 52-х років скаржився на раптову слабкість і оніміння в лівих кінцівках, утруднення при ходьбі. Об’єктивно відзначалися лівобічні гемігіпестезія і легкий геміпарез. Через 4 години стан хворого нормалізувався, вогнищева симптоматика регресувала, хворий зміг нормально ходити. AT-120/80 мм рт.ст. Який найбільш імовірний діагноз? A 52-year-old patient complained of sudden weakness and numbness in the left limbs, difficulty walking. Objectively, left-sided hemipaesthesia and mild hemiparesis were noted. After 4 hours, the condition the patient recovered, focal symptoms regressed, the patient was able to walk normally. AT-120/80 mm Hg. What is the most likely diagnosis?

Транзиторна ішемічна атака Transient ischemic attack

Геморагічний інсульт Hemorrhagic stroke

Асоційована мігрень Associated Migraine

Ішемічний інсульт Ischemic stroke

Гіпертеязивний криз Hypertensive crisis